Final Exam Flashcards

1
Q

What is the definition of “pathophysiology”?

A

Pathophysiology is defined as not only the cellular and organ changes that occur with disease, but also the effects that these changes have on total body function.

How well did you know this?
1
Not at all
2
3
4
5
Perfectly
2
Q

Name the 5 etiologic factors and give an example if listed.

A

Biologic agents (bacteria, viruses), physical forces (trauma, burns, radiation), chemical agents (poisons, alcohol), one’s genetic inheritance, and nutritional excesses or deficiencies.

How well did you know this?
1
Not at all
2
3
4
5
Perfectly
3
Q

What are the 2 types of risk factors?

A

Risk factors can be categorized as congenital conditions (present at birth) or acquired defects (occurring after birth).

How well did you know this?
1
Not at all
2
3
4
5
Perfectly
4
Q

What is the difference between morphology and histology?

A

Morphology is defined as the fundamental structure or form of cells or tissues. Histology is the study of the cells and extracellular matrix of body tissues.

How well did you know this?
1
Not at all
2
3
4
5
Perfectly
5
Q

Describe the difference between signs and symptoms and give an example of each.

A

A symptom is a subjective complaint, for example pain, trouble breathing, or dizziness. A sign is an objective manifestation, for example an elevated temperature, a swollen extremity, or changes in pupil size.

How well did you know this?
1
Not at all
2
3
4
5
Perfectly
6
Q

What are 3 important processes when coming to a diagnosis?

A

The diagnostic process requires a careful history, physical examination (PE), and sometimes diagnostic tests.

How well did you know this?
1
Not at all
2
3
4
5
Perfectly
7
Q

Explain validity, reliability, sensitivity, and specificity.

A

Validity is how a tool measures what it is intended to measure. Reliability is how likely the same result will occur if repeated. Sensitivity is the proportion of people with a disease who are positive for that disease. Specificity are people without the disease who are negative on a given test.

How well did you know this?
1
Not at all
2
3
4
5
Perfectly
8
Q

Define epidemiology and name some things that it tracks.

A

Epidemiology is the study of disease occurrence in human populations. It tracks age, race, dietary habits, lifestyle, or geographic location.

How well did you know this?
1
Not at all
2
3
4
5
Perfectly
9
Q

What is the difference between incidence and prevalence?

A

Incidence is the number of new cases in a population at risk during a specified time. Prevalence is the number of people with the disease in a population in a given time.

How well did you know this?
1
Not at all
2
3
4
5
Perfectly
10
Q

Define mortality and morbidity.

A

Morbidity is the effect of an illness on one’s life. Mortality statistics deal with the cause of death in a population.

How well did you know this?
1
Not at all
2
3
4
5
Perfectly
11
Q

Define primary, secondary, and tertiary prevention and give an example of each.

A

Primary prevention is to remove risk factors to prevent disease from occurring- taking folic acid while pregnant to prevent neural tube defects, vaccinating children to prevent communicable disease, eating healthy and exercising to prevent heart disease, and wearing seatbelts or helmets. Secondary prevention aims to detect and treat disease early, usually while the disease is asymptomatic and curable- annual Pap smears to detect early cervical cancer, encouraging smoking cessation, checking blood pressure and cholesterol, and colonoscopy screening. Tertiary prevention occurs after a disease has been diagnosed and clinical intervention is needed to reduce complications or deterioration- certain medications one must take after a heart attack to help reduce the risk of a future event or death.

How well did you know this?
1
Not at all
2
3
4
5
Perfectly
12
Q

Why is evidence-based practice important?

A

Evidence-based practice is the conscientious, explicit, and judicious use of current best evidence in making decisions about the care of individual patients. This is to counteract “the way things have always been done,” and to practice based on clinical research.

How well did you know this?
1
Not at all
2
3
4
5
Perfectly
13
Q

What are the 3 primary structures of the cell?

A

All eukaryotic cells have three primary structures that are considered the functional components of the cell. These are the nucleus, the cytoplasm, and the plasma membrane.

How well did you know this?
1
Not at all
2
3
4
5
Perfectly
14
Q

What is the function of the nucleus?

A

The nucleus is the control center for the cell and contains most of the hereditary material, DNA and RNA.

How well did you know this?
1
Not at all
2
3
4
5
Perfectly
15
Q

Name 3 structures within the nucleus.

A

Chromatin, nucleolus, and nuclear envelope

How well did you know this?
1
Not at all
2
3
4
5
Perfectly
16
Q

What organelles are located in the cytoplasm?

A

The organelles in the cytoplasm include the ribosomes, ER, Golgi complex, mitochondria, and lysosomes.

How well did you know this?
1
Not at all
2
3
4
5
Perfectly
17
Q

What are the roles of the ribosomes, rough ER, and smooth ER?

A

The ribosomes are the site of protein synthesis in the cell. The Rough ER has ribosomes and functions to synthesize lysosomal enzymes. The smooth ER does not have ribosomes and is the site for lipid, lipoprotein, and steroid hormone synthesis.

How well did you know this?
1
Not at all
2
3
4
5
Perfectly
18
Q

What is the function of the lysosomes, peroxisomes, and mitochondria?

A

Lysosomes are the digestive system of the cell. Peroxisomes are smaller than lysosomes and function in the control of free radicals. The mitochondria are aptly termed the “power plants” of the cell, as they transform organic compounds (energy in food) into cellular energy for the cell.

How well did you know this?
1
Not at all
2
3
4
5
Perfectly
19
Q

What is contained in the cytoskeleton and what is its function?

A

The cytoskeleton contains a network of microtubules, microfilaments, intermediate filaments, and thick filaments. They control cell shape and movement.

How well did you know this?
1
Not at all
2
3
4
5
Perfectly
20
Q

Give examples of where cilia and flagella are found in the human body.

A

Cilia can be found on many epithelial linings, including the nasal sinuses and bronchi in the upper respiratory system. In the human body, only spermatozoa contain flagella.

How well did you know this?
1
Not at all
2
3
4
5
Perfectly
21
Q

What are the 4 functions of the cell membrane?

A

The cell membrane controls the transport of materials from the outside fluids to within, binds hormone receptors, helps with the conduction of electrical currents in nerve and muscle cells, and aids in the regulation of cell growth and proliferation.

How well did you know this?
1
Not at all
2
3
4
5
Perfectly
22
Q

Why do cells need to communicate with one another?

A

In complex organisms, cells develop special functions, contribute to growth, and adapt to changes. Cells must have the ability to communicate with one another, transport substances in and out, and respond to these changes.

How well did you know this?
1
Not at all
2
3
4
5
Perfectly
23
Q

Name the 4 types of cell signaling.

A

Endocrine signaling - hormones carried in the bloodstream to act on cells throughout the body.

Paracrine signaling - enzymes metabolize chemical mediators, producing a change in neighboring cells.

Autocrine signaling - a cell releases a chemical into the extracellular fluid that affects its own activity.

Synaptic signaling - occurs in the nervous system as neurotransmitters act on adjacent nerve cells through synapses.

How well did you know this?
1
Not at all
2
3
4
5
Perfectly
24
Q

Know the process of up and down regulation.

A

Every cell has a distinct set of surface receptors that allow it to respond to signaling molecules in a specific way. These proteins can increase or decrease in number according to the needs of the cell. When there are too many chemical messengers present, the number of active receptors decreases in a process called down-regulation. When the messengers are lacking, the number of active receptors increases through up-regulation.

How well did you know this?
1
Not at all
2
3
4
5
Perfectly
25
Q

Explain diffusion, facilitated diffusion, and active transport.

A

Diffusion is the process by which electrolytes move from an area of higher concentration on one side of the membrane to an area of lower concentration on the other side. Facilitated diffusion uses a transport protein to help lipid insoluble or large molecules pass through the membrane, that otherwise would not be able to get through. Active transport is used when cells use energy to move ions against an electrical or chemical gradient.

How well did you know this?
1
Not at all
2
3
4
5
Perfectly
26
Q

What is the most important active transport system and why is it important?

A

The most studied active transport system is the sodium-potassium (Na+/K+)-ATPase pump. The pump moves sodium from inside the cell to the extracellular region and returns potassium to the inside of the cell. If this process didn’t occur, sodium would remain in the cell and water would follow, causing the cell to swell.

How well did you know this?
1
Not at all
2
3
4
5
Perfectly
27
Q

Define endocytosis and the 2 processes it encompasses.

A

Endocytosis is when a molecule outside of a cell is enclosed in an invagination of the cell membrane, forming a vesicle inside the cell. Pinocytosis (“cell drinking”) engulfs small solid or fluid particles like proteins and electrolytes. Phagocytosis (“cell eating”) is when the membrane engulfs and then kills microorganisms or other particulate matter. When the particle is enclosed, a phagosome is formed which is moved into the cytoplasm, where it joins with a lysosome to be destroyed.

How well did you know this?
1
Not at all
2
3
4
5
Perfectly
28
Q

Why are ion channels necessary?

A

Because small ions like sodium and potassium have an electrical charge, they are unable to get through the lipid layer of the cell membrane. Their way across is by facilitated diffusion through selective ion channels.

How well did you know this?
1
Not at all
2
3
4
5
Perfectly
29
Q

Understand the different types of transport systems.

A

n primary active transport, ATP is used directly to transport the substance. In secondary active transport, energy is derived from the primary active transport of one substance, usually sodium, for the cotransport of a second substance. When sodium leaves the cell by primary active transport, a concentration gradient develops. This results in storage of energy because sodium is wanting to diffuse back into the cell. Two groups of secondary active transport are available. Cotransport or symport systems consist of sodium ions and the solute being transported in the same direction. Counter-transport or antiport systems has sodium ions and the solute being transported in the opposite direction. An example of cotransport occurs in the intestine in which the absorption of glucose and amino acids is paired with sodium transport.

How well did you know this?
1
Not at all
2
3
4
5
Perfectly
30
Q

What are membrane potentials?

A

Electrical potentials that exist in the cell membrane are called membrane potentials. Membrane potentials in nerve and muscle cells are needed to generate nerve impulses and muscle contractions. In other cells, changes in the membrane potential can cause hormone secretion.

How well did you know this?
1
Not at all
2
3
4
5
Perfectly
31
Q

Be familiar with the 4 different types of tissue and characteristics of each.

A

Epithelial tissue covers the body’s outer surface, lines the inner surfaces, and forms glandular tissue. The basal surface is attached to an underlying basement membrane, it is avascular and draws oxygen and nutrients from the capillaries of the connective tissue on which it rests. Epithelial tissues can be squamous, cuboidal, and columnar. They can be arranged in simple, stratified, and pseudostratified layers.

Connective tissues produce extracellular matrix that supports and holds tissues together. Connective tissue proper include loose (areolar), adipose, reticular, and dense connective tissue. Specialized connective tissue includes cartilage, bone, and blood cells.

Muscle tissues move the skeletal structures, pump blood through the heart, and contract the blood vessels and visceral organs. THe fibers that produce muscle tissue contraction are thin filaments called actin and thick filaments called myosin. Muscle tissue can be skeletal, cardiac, or smooth.

Nervous tissue is used for communication between peripheral tissues and the central nervous system. It provides the means for controlling body function and for sensing and moving about the environment in response to internal and external stimuli. The two types of nervous tissue cells are neurons (communication) and glial cells (support the neurons).

How well did you know this?
1
Not at all
2
3
4
5
Perfectly
32
Q

What is atrophy and what causes it?

A

Atrophy is a decrease in the size of an organ or tissue resulting from a decrease in the mass of pre-existing cells. It’s typically a result from disuse, nutritional or oxygen deprivation, diminished endocrine stimulation, aging, and denervation (lack of nerve stimulation in peripheral muscles caused by injury to motor nerves).

How well did you know this?
1
Not at all
2
3
4
5
Perfectly
33
Q

Know the difference between hypertrophy and hyperplasia.

A

Hypertrophy is the increase in the size of an organ or tissue due to the increase in the size of the cells that comprise it, often related to workload.

Hyperplasia is the increase in the size of an organ or tissue caused by an increase in the number of cells making up that organ or tissue.

How well did you know this?
1
Not at all
2
3
4
5
Perfectly
34
Q

What is the advantage and disadvantage of metaplasia?

A

Metaplasia usually occurs in response to chronic irritation and inflammation and allows for substitution of cells that are better able to survive under those circumstances. Under persistent stress, it can progress to dysplasia.

How well did you know this?
1
Not at all
2
3
4
5
Perfectly
35
Q

Define dysplasia. Is it reversible? What can it lead to?

A

Dysplasia is disordered cellular growth and can result in cells that vary in size, shape, and organization. Dysplasia is reversible, in theory, with alleviation of inciting stress. If stress persists, dysplasia progresses to carcinoma (irreversible).

How well did you know this?
1
Not at all
2
3
4
5
Perfectly
36
Q

List the 5 categories in which cells can be damaged and give an example of each.

A

Cells can be damaged by the following: (1) Injury from physical agents (mechanical forces [fractures], extremes of temperature [burns], and electrical forces); (2) Radiation injury (ionizing [cancer treatment], ultraviolet [sunburn], or nonionizing [thermal burns] radiation.); (3) Chemical injury (drugs [alcohol, Rx/OTC drugs, street drugs], carbon tetrachloride, lead toxicity, and mercury toxicity.); (4) Injury from biologic agents (viruses, parasites, and bacteria); and (5) Injury from nutritional imbalances (excess or a deficiency).

How well did you know this?
1
Not at all
2
3
4
5
Perfectly
37
Q

What is apoptosis?

A

Apoptosis is programmed cell death that eliminates cells that are worn out, have been produced in excess, developed improperly, or are genetically damaged.

How well did you know this?
1
Not at all
2
3
4
5
Perfectly
38
Q

Know the different types of necrosis.

A

Coagulative necrosis is often from a sudden cutoff of blood supply to an organ. Liquefactive necrosis occurs when some of the cells die but their catalytic enzymes are not destroyed. Caseous necrosis is part of granulomatous inflammation and often associated with tuberculosis. Gangrenous necrosis is when a significant amount of tissue undergoes necrosis and can be characterized as either dry or moist.

How well did you know this?
1
Not at all
2
3
4
5
Perfectly
39
Q

What are the three types of cell injury?

A

Free radical formation - free radicals are highly reaction making them unstable, enabling them to disrupt and damage cells and tissues.

Hypoxia - hypoxic cell injury is due to low oxygen delivery to the tissues. Decreased oxygen impairs oxidative metabolism, resulting in the decreased production of ATP (energy), which leads to cellular injury as necessary metabolic processes cannot be efficiently carried out.

Disruption of intracellular calcium homeostasis - calcium functions as a second messenger for many cell reactions. Usually intracellular calcium levels are lower than extracellular levels, thus a rise in intracellular calcium can inappropriately activate enzymes leading to potentially damaging effects.

How well did you know this?
1
Not at all
2
3
4
5
Perfectly
40
Q

During pregnancy, uterine enlargement is caused by what?

A

Hypertrophy and hyperplasia.

How well did you know this?
1
Not at all
2
3
4
5
Perfectly
41
Q

Persistent dysplasia eventually results in….

A

cancer.

How well did you know this?
1
Not at all
2
3
4
5
Perfectly
42
Q

Is endometrial hyperplasia a normal physiologic occurrence?

A

No

How well did you know this?
1
Not at all
2
3
4
5
Perfectly
43
Q

A patient complains of chest pain and an elevated blood pressure. What are these examples of?

A

Signs & symptoms

How well did you know this?
1
Not at all
2
3
4
5
Perfectly
44
Q

Explain apoptosis and why it is necessary:

A

Apoptosis is programmed cell death. This process eliminates cells that are worn out, have been produced in excess, have developed improperly, or have genetic damage. Apoptosis is also responsible for several normal physiologic processes, like replacing cell in the intestinal villi and removing aging red blood cells.

How well did you know this?
1
Not at all
2
3
4
5
Perfectly
45
Q

Explain what necrosis is and give an example and description of one type of necrosis.

A

Necrosis refers to cell death in an organ or tissues that is still part of a living person. It often interferes with cell replacement and tissue regeneration. Coagulative necrosis results most often from a sudden cutoff of blood supply to an organ (ischemia), particularly the heart and kidney. Liquefactive necrosis occurs when some of the cells die but their catalytic enzymes are not destroyed. It is commonly seen with brain infarcts or abscesses. Caseous necrosis occurs as part of granulomatous inflammation and is most often associated with tuberculosis.

Gangrenous necrosis most often affects the lower extremities or bowel and is secondary to vascular occlusion. The term gangrene is applied when a considerable mass of tissue undergoes necrosis. In dry gangrene the affected tissue becomes dry and shrinks, the skin wrinkles, and its color changes to dark brown or black. The spread of dry gangrene is slow. It results from a cut off in arterial blood supply and is a form of coagulation necrosis. In wet gangrene, the affected area is cold, swollen, and pulseless. The skin is moist, black, and under tension. Blebs form on the surface, liquefaction occurs, and a foul odor is caused by bacterial action. The spread of tissue damage is rapid.

How well did you know this?
1
Not at all
2
3
4
5
Perfectly
46
Q

List the 4 types of tissue found in the body. Pick 2 and give a description and example of each.

A

Epithelial tissue covers the body’s outer surface, lines the inner surfaces, and forms glandular tissue. Epithelial tissue has three distinct surfaces and the basal surface is attached to an underlying basement membrane. It is avascular, meaning without blood vessels. It receives oxygen and nutrients from the capillaries of the connective tissue on which it rests.

Connective or supportive tissue is the most abundant tissue in the body. It connects and binds or supports the various tissues. Its cells produce the extracellular matrix that support and hold tissues together. Connective tissue is divided into two types: connective tissue proper and specialized connective tissue (cartilage, bone, and blood cells). The four types of connective tissue proper are loose (areolar), adipose, reticular, and dense connective tissue.

The function of muscle tissue is to move the skeletal structures, pump blood through the heart, and contract the blood vessels and visceral organs. Muscle tissue can accomplish this by contraction. The two types of fibers that contract are called thin and thick filaments. Thin filaments are called actin, and the thick filaments are myosin. The three types of muscles tissue are skeletal, cardiac, and smooth.

Nervous tissue is distributed throughout the body for communication. It provides the means for controlling body function and for sensing and moving about the environment. The two types of cells are neuron and glial cells. Neurons function is communication. Glial (meaning glue) cells support the neurons.

How well did you know this?
1
Not at all
2
3
4
5
Perfectly
47
Q

What is the most studied active transport system in the human body?

A

The most studied active transport system in the human body is the sodium-potassium-ATPase active transport system that moves sodium out of the cell and potassium into the cell to prevent cells from swelling too much from water.

How well did you know this?
1
Not at all
2
3
4
5
Perfectly
48
Q

What term means “cell drinking,” and engulfs small solid or fluid particles, as seen with proteins and electrolytes?

A

Pinocytosis.

How well did you know this?
1
Not at all
2
3
4
5
Perfectly
49
Q

Give one function of a membrane potential:

A

Generate nerve impulse, muscle contractions, or cause hormone secretion

How well did you know this?
1
Not at all
2
3
4
5
Perfectly
50
Q

What is the term that best describes the following process?
A transport protein aiding a lipid insoluble or large molecule across the cell membrane that would otherwise not be able to pass through on its own.

A

facilitated diffusion

How well did you know this?
1
Not at all
2
3
4
5
Perfectly
51
Q

Define neoplasia and neoplasm:

A

Neoplasia is the process of unregulated growth of abnormal cells with patterns of altered cell differentiation and growth. A neoplasm is the new growth itself.

How well did you know this?
1
Not at all
2
3
4
5
Perfectly
52
Q

Define proliferation and differentiation:

A

Proliferation, a process of cell division, is an adaptive process for new cell growth to replace old cells or when additional cells are needed. Differentiation is the process in which cells become more specialized with each mitotic division.

How well did you know this?
1
Not at all
2
3
4
5
Perfectly
53
Q

What are the 3 main groups of cells that proliferate?

A

(1) well-differentiated neurons and cells of skeletal and cardiac muscles that rarely divide and reproduce; (2) progenitor or parent cells that continue to divide and reproduce, like blood, skin, and liver cells; (3) undifferentiated stem cells that can enter the cell cycle and produce large number of progenitor cells if needed.

How well did you know this?
1
Not at all
2
3
4
5
Perfectly
54
Q

What are progenitor cells?

A

The more specialized cells are unable to divide, so these cell populations rely on progenitor or parent cells of the same lineage that have not yet differentiated to the extent that they have lost their ability to divide. They have enough differentiation so the daughter cells are limited to the same cell line, but not quite differentiated enough to prohibit active proliferation.

How well did you know this?
1
Not at all
2
3
4
5
Perfectly
55
Q

What are the benefits of the stem cell?

A

Stem cells remain incompletely differentiated and dormant until they are needed. When they divide they produce other stem cells, but also cells capable of carrying out the functions of the needed differentiated cell (i.e. progenitor cells) until they reach a state of terminal differentiation.

Stem cells possess self-renewal, meaning they can undergo numerous mitotic divisions while maintaining an undifferentiated state, and they have different levels of potency - they can differentiate into different states.

How well did you know this?
1
Not at all
2
3
4
5
Perfectly
56
Q

What is a polyp?

A

A polyp is a growth that projects from a mucosal surface, such as the intestine. A polyp can be benign or malignant.

How well did you know this?
1
Not at all
2
3
4
5
Perfectly
57
Q

Explain metastasis and how it occurs:

A

Metastasis is when a secondary tumor develops in a distant location from the primary tumor. It occurs via the lymph channels and blood vessels. The cancer cell is shed from the primary tumor, it invades the surrounding extracellular matrix where it gains access to a blood vessel. If the cells survive the bloodstream, it has to invade tissue elsewhere, grow, and establish a blood supply as well as growth factors.

How well did you know this?
1
Not at all
2
3
4
5
Perfectly
58
Q

What are the two broad categories of malignant neoplasms?

A

Solid tumors and hematologic cancers

How well did you know this?
1
Not at all
2
3
4
5
Perfectly
59
Q

Explain what carcinoma in situ is?

A

Carcinoma in situ is a localized preinvasive lesion. These can typically be surgically removed or treated, and recurrence is less likely.

How well did you know this?
1
Not at all
2
3
4
5
Perfectly
60
Q

What is seeding?

A

Seeding is the process of how shed tumor cells enter circulation and move into similar or different body cavities. It often happens in the peritoneal cavity. It usually causes fluid accumulation.

How well did you know this?
1
Not at all
2
3
4
5
Perfectly
61
Q

What is important about the sentinel node?

A

The sentinel node is the first lymph node to which the primary tumor drains. Through lymphatic mapping and sentinel lymph node biopsy, the extent of the disease can be determined.

How well did you know this?
1
Not at all
2
3
4
5
Perfectly
62
Q

What is angiogenesis?

A

When the tumor reaches the distant site, it must establish blood vessels and growth factors to continue to grow. The development of new blood vessels within the tumor is termed angiogenesis.

How well did you know this?
1
Not at all
2
3
4
5
Perfectly
63
Q

What are the 2 broad etiologic causes of cancer?

A

The genetic and molecular mechanisms that transform normal cells into cancer cells.

The external factors such as age, heredity, and environmental agents.

How well did you know this?
1
Not at all
2
3
4
5
Perfectly
64
Q

What are the 2 cancer associated genes?

A

Proto-onocogenes which are normal genes that become cancer causing oncogenes if mutated. They are associated with gene overactivity.

Tumor suppressor genes slow down cell division, repair DNA mistakes, or tell cells when to die. They are associated with gene underactivity.

How well did you know this?
1
Not at all
2
3
4
5
Perfectly
65
Q

What are the 3 genetic events that lead to oncogene formation and activation?

A

Point mutation - there is a single nucleotide base change due to an insertion, deletion, or substitution.

Chromosomal translocation - the unusual movement or rearrangement of a segment of chromosomal DNA.

Gene amplification - the unusual increase in the number of copies of a certain gene. This leads to an unwanted over-expression of the gene. As the amount of proteins encoded by the gene drastically increases, the result is often an increase in cell proliferation.

How well did you know this?
1
Not at all
2
3
4
5
Perfectly
66
Q

What the 6 molecular and cellular mechanisms that are known to facilitate the development of cancer?

A

Genes that increase susceptibility to cancer include defects in DNA repair mechanisms, defects in growth factor signaling pathways, evasion of apoptosis, avoidance of cellular senescence, development of sustained angiogenesis, and metastasis and invasion.

How well did you know this?
1
Not at all
2
3
4
5
Perfectly
67
Q

What are the 7 risk factors that can lead to the development of cancer?

A

Risk factors linked to cancer include heredity, hormonal factors, obesity, immunologic mechanisms, environmental agents such as chemicals, radiation, and cancer-causing viruses.

How well did you know this?
1
Not at all
2
3
4
5
Perfectly
68
Q

Give an example of a hereditary cancer:

A

Breast cancer has been linked with BRCA1 and 2.

How well did you know this?
1
Not at all
2
3
4
5
Perfectly
69
Q

What organs do hormones and the development of cancer target?

A

Breast, ovary, endometrium, prostate

How well did you know this?
1
Not at all
2
3
4
5
Perfectly
70
Q

List some lifestyle factors that contribute to the development of cancer?

A

Smoking, alcohol consumption, eating red meat and meats that have been cooked over charcoal, low consumption of plants.

How well did you know this?
1
Not at all
2
3
4
5
Perfectly
71
Q

What are the 4 oncogenic viruses that can induce cancer?

A

Human Papilloma Virus (HPV) –> cervical cancer

EBV –> Burkitt lymphoma, nasopharyngeal cancer, B Cell lymphomas, and Hodgkin lymphoma

HHV-8 –> Kaposi Sarcoma

HBV –> liver cancer

How well did you know this?
1
Not at all
2
3
4
5
Perfectly
72
Q

List some common symptoms people with cancer have:

A

Bleeding, pleural/peritoneal fluid, anorexia, weight loss, wasting of body fat and muscle tissue, weakness, fatigue, sleep disturbances, anemia.

How well did you know this?
1
Not at all
2
3
4
5
Perfectly
73
Q

Why is anemia common in people with cancer?

A

Anemia is common because of the effects of the treatment or because of blood loss, hemolysis, or impaired red blood cell production.

How well did you know this?
1
Not at all
2
3
4
5
Perfectly
74
Q

What are paraneoplastic syndromes? Give an example.

A

Cancer can also produce symptoms in sites not directly affected by the disease. These manifestations are termed paraneoplastic syndromes. Examples are syndrome of inappropriate ADH secretion, Cushing syndrome due to ectopic ACTH production, and hypercalcemia.

How well did you know this?
1
Not at all
2
3
4
5
Perfectly
75
Q

What body parts do we screen through observation? Palpation? Laboratory tests and procedures?

A

Observation (skin, mouth, external genitalia), palpation (breast, thyroid, rectum and anus, prostate, lymph nodes), and laboratory tests and procedures (Pap smear, colonoscopy, mammography)

How well did you know this?
1
Not at all
2
3
4
5
Perfectly
76
Q

What are tumor markers helpful for?

A

Establishing prognosis, monitoring treatment, and detecting recurrent disease.

How well did you know this?
1
Not at all
2
3
4
5
Perfectly
77
Q

What is the most important procedure in diagnosing the correct cancer and histology?

A

Tissue biopsy

How well did you know this?
1
Not at all
2
3
4
5
Perfectly
78
Q

Define grading and staging and be familiar with table 2.5.

A

Grading is based on the cellular characteristics of the tumor and the degree of abnormalities present; it involves the microscopic examination of cancer cells to determine their level of differentiation.

Staging is the assessment of the clinical spread of the disease.

How well did you know this?
1
Not at all
2
3
4
5
Perfectly
79
Q

What is the purpose of grading and staging a tumor?

A

Both methods are used to determine the course of the disease and aid in selecting an appropriate treatment or management plan.

How well did you know this?
1
Not at all
2
3
4
5
Perfectly
80
Q

What are the 3 possible goals of cancer treatment?

A

Curative, control, and palliative

How well did you know this?
1
Not at all
2
3
4
5
Perfectly
81
Q

When is surgery appropriate in the treatment of cancer?

A

Surgery is appropriate to remove cancer, for diagnosing, staging, and palliation. This is determined by the extent of the disease, location, structures involved, tumor growth rate, and quality of life for the patient post-surgery.

If the tumor is small with well-defined margins; to treat oncologic emergencies; prophylactically

How well did you know this?
1
Not at all
2
3
4
5
Perfectly
82
Q

How does radiation kill cancer cells?

A

Radiation uses high energy particles or waves to destroy or damage cancer cells. It leads to the creation of free radicals which damage cell structures. It can interrupt the cell cycle process, kill cells, or damage DNA in the cells. It impacts all proliferating cells, but normal tissue is usually able to recover from the damage more readily than cancerous tissue.

How well did you know this?
1
Not at all
2
3
4
5
Perfectly
83
Q

List some side effects of chemotherapy drugs.

A

Neutropenia (risk for infections), anemia (Causing fatigue), thrombocytopenia (risk for bleeding)m anorexia, nausea, vomiting, alopecia.

How well did you know this?
1
Not at all
2
3
4
5
Perfectly
84
Q

List the 5 possible treatments for cancer:

A

Surgery, radiation therapy, chemotherapy, hormonal therapy, biotherapy.

How well did you know this?
1
Not at all
2
3
4
5
Perfectly
85
Q

Describe the cell cycle

A

The cell cycle is the process by which a cell duplicates its genetic information and divides between two genetically identical daugher cells. In Gap 1 - DNA synthesis stops while the cell enlarges and RNA and protein synthesis begins. S phase - DNA synthesis occurs, producing two separate sets of chromosomes, one for each daughter cell. Gap 2 - DNA synthesis stops again and RNA/protein synthesis continues. G1, S, G2 - interphase.
Between each phase are cell cycle checkpoints to ensure the cell is ready to proceed to the next phase, and if not, the cycle is halted and allowed to complete its replication or repair DNA damage if detected, ensuring all genetic information is passed on correctly. M Phase - mitosis (dividing the DNA) and cytoplasmic division.

Continually dividing cells (squamous epithelium) cycle from one mitotic division to the next, while some cells go into a resting state G0 if nutrients or growth factors are unavailable or highly specialized cells first leave the cell cycle. Cells in G0 may re-entre the cell cycle when nutrients become available. Highly specialized or terminally differentiated cells like neurons may stay permanently in G0.

How well did you know this?
1
Not at all
2
3
4
5
Perfectly
86
Q

What is the difference between benign and malignant tumors (table 2.2)

A

Benign cells are differentiated and similar to existing tissue; the growth rate is slow - it can stop growing or regress; the cells are non-invasive and usually encapsulated; they do not metastasize; and, they have a low mortality rate.

Malignant tumor cells are undifferentiated with atypical structure; the growth rate is variable - increased undifferentiation leads to more rapid growth rate; the cell growth is invasive and infiltrates surrounding tissues; the tumors are metastatic and use blood and lymphatic vessels to travel to distant sites; and they are associated with a high mortality.

How well did you know this?
1
Not at all
2
3
4
5
Perfectly
87
Q

Describe cancer cell characteristics (table 2.3)

A

Anaplasia - loss of cell differentiation in cancerous tissue. Highly anaplastic cells resemble undifferentiated or embryonic cells, rather than the tissue of origin.
Genetic instability - under normal conditions, cells are protected from genetic errors because of the many cellular mechanisms in place to prevent them. The dysregulated nature of cancer leads to a high frequency of genetic errors and therefore genetic instability -.
Growth factor independence - cancer cells can proliferate deven in the absence of growth factors
Cell density-dependent inhibition - the cessation of growth after cells reach a certain density is lost in cancer cells.
Cell cohesiveness and adhesion - cells do not stick together which permits surface cells of the tumor to shed into the surrounding body fluids or secretions.
Anchorage dependence - cancer cells can remain viable and multiply without being anchored to neighboring cells or the underlying extracellular matrix.
Cell to cell communication - diminished in cancer cells which interferes with intercellular connections and the responsiveness to membrane-derived signals.
Unlimited lifespan - Cancer cells can divide an infinite number of times. Telomeres, the protective ‘end caps’ on chromosomes, typically shorten with each cell division. Most cancer cells keep high levels of telomerase - an enzyme that prevents telomere shortening. Older and more error prone cells continue to replicate giving rise to cell populations of increasing levels of dysfunction.
Antigen expression - cancer cells contain several antigens that are immunologically different from its normal tissue counterpart (useful as identification markers)
Production of enzymes, hormones, etc - unlike tissues of origin. Can secrete hormones or enzymes that promote metastasis.
Cytoskeletal changes or abnormalities - abnormal intermediate filament types or changes in actin filaments and microtubules to enhance their ability for invasion and meatstasis.

How well did you know this?
1
Not at all
2
3
4
5
Perfectly
88
Q

Explain the TNM system

A

The TNM system is an advancement of the Staging system to determine the spread of cancer in a patient. It looks at the stages of advancement in the tumors, lymph notes, and metastasis in a patient. It looks to describe if tumors have spread, if the cancer is in more than one lymph node, and the degree of metastasis. It’s a more specific and holisitc staging system. It does use a slightly different numbering system than the normal stages of I through IV. For example, tumors (T) can be rated at Tx, T0, T1, T2, etc.

How well did you know this?
1
Not at all
2
3
4
5
Perfectly
89
Q

When would surgery be appropriate in the treatment of cancer?

A

Surgery is often the first treatment for solid tumors. If the tumor is small with well-defined margins, it can be removed completely. It is also used for oncologic emergencies and prophylactic surgery in high risk patients.

How well did you know this?
1
Not at all
2
3
4
5
Perfectly
90
Q

Most chemotherapeutic drugs cause pancytopenia due to bone marrow suppression. What are the 3 possible adverse outcomes of this?

A

Neutropenia- risk for infections

Anemia- causing fatigue

Thrombocytopenia- risk for bleeding

How well did you know this?
1
Not at all
2
3
4
5
Perfectly
91
Q

How do cancer cells achieve immortality?

A

Cancer cells keep high levels of telomerase, an enzyme that prevents telomere shortening. This gives the ability of the chromosomes to continue to replicate forever.

How well did you know this?
1
Not at all
2
3
4
5
Perfectly
92
Q

What is a tumor suppressor gene? Give one example.

A

A tumor suppressor gene encodes proteins to slow the growth of cells and trigger apoptosis when necessary. When the tumor suppressor gene is suppressed as a result of neoplasia, it can allow cancerous cells to proliferate at a faster rate. BRCA1 or 2, TP53

How well did you know this?
1
Not at all
2
3
4
5
Perfectly
93
Q

A 40-year-old woman has experienced heavy menstrual bleeding. She was told she has a uterine tumor called a leiomyoma. She is worried she has cancer. What do you tell her? Explain at least 2 differences between a benign and malignant tumor.

A

Leiomyoma is a benign tumor. (Leiomyosarcoma is malignant) Student can add any of the following: Benign tumors are well-differentiated cells, resemble the cells of tissues of origin, and have a slow, progressive rate of growth. They grow by expansion and remain localized to their site of origin, not capable of metastasizing. They develop a rim of connective tissue around the tumor called a fibrous capsule, which aids in surgical removal. Benign tumors are less of a threat unless they interfere with vital functions

Malignant neoplasms invade and destroy tissue. They grow rapidly, spread to other parts of the body, and lack well-defined margins. They can compress blood vessels and outgrow their blood supply, causing ischemia and tissue injury. Surgery can be more difficult if it has spread.

How well did you know this?
1
Not at all
2
3
4
5
Perfectly
94
Q

A 50-year-old woman was diagnosed with breast cancer from tissue biopsy following her diagnostic mammography. They are determining if it has spread to the lymph nodes. (1) Explain what the sentinel node is. (2) How will they determine if it has spread to the lymph system? (3) Would a detailed family history of disease be important to obtain?

A

The sentinel node is the first lymph node to which the primary tumor drains. The extent of disease may be determined through lymphatic mapping and sentinel lymph node biopsy. A radioactive tracer and/or blue dye is injected into the tumor to determine the first lymph node. This lymph node is examined for the presence of cancer cells. Yes, heredity can be a strong risk factor for cancer in some families.

How well did you know this?
1
Not at all
2
3
4
5
Perfectly
95
Q

What is innate immunity

A

Innate (natural) immunity is the first line of defense in the body’s immune system. There are physical barriers that are part of the innate immunity to prevent foreign microorganisms to enter the body - this can include skin which has closely packed cells in multiple layers as well as a layer of Keratin that creates a salty, acidic environment inhospitable to microbes. The skin also has antimicrobial proteins and lysozymes that inhibit microorganisms and help to destroy them. Additionally the epithelial cells that line and protect the GI, respiratory, and urogenital tract.

In addition to they physical, innate immunity includes cellular defenses in the forms of neutrophils, macrophages, dendritic cells, natural killer cells, and intraepithelial lymphocytes - and they respond to foreign particles.

How well did you know this?
1
Not at all
2
3
4
5
Perfectly
96
Q

How does the skin act as a physical barrier?

A

The skin’s design makes it a strong physical barrier. It has closely packed cells in multiple layers that are continuously being shed. Keratin covers the skin, which creates a salty, acidic environment inhospitable to microbes. It also contains antimicrobial proteins and lysozymes that inhibit microorganisms and help to destroy them.

How well did you know this?
1
Not at all
2
3
4
5
Perfectly
97
Q

What cells are involved in innate immunity?

A

They are divided into two categories: granulocytes (contain granules - neutrophils, eosinophils, and basophils) and agranulocytes (do not contain granules - monocytes, macrophages, lymphocytes).

Neutrophils (most abundant in the body, 55% of white blood cells) are early responders in innate immunity and use phagocytosis to kill microbes.

Eosinophils (1-4%) are active in parasitic infections and allergic responses.

Basophils (<1%) release histamine and proteolytic enzymes.

Monocytes (3-7%) are the largest in size of the leukocytes. They are released from bone marrow and mature into macrophages and dendritic cells where they engage in the inflammatory response and phagocytize foreign substances and cellular debris.

Macrophages have a longer life span, reside in the tissues, and are the first phagocyte that organisms encounter. Neutrophils and macrophages work together on behalf of the host’s initial defense system.

Natural Killer Cells and Intraepithelial cells are both involved in the innate immune response. Natural killer cells can spontaneously kill target organisms, some types of tumors and infected cells without previous exposure to surface antigens. They can also limit the spread of infection and assist in the development of adaptive immune responses through cytokine production. NK cells help with dendritic cell maturation and innate immune control of viral infections.

Dendritic cells are immature cells in lymphoid tissue where they capture foreign agents and transport them to peripheral lymphoid organs. Once they are activated, they mature and migrate to regional lymph nodes. They are key antigen-presenting cells capable of initiating adaptive immunity.

How well did you know this?
1
Not at all
2
3
4
5
Perfectly
98
Q

Describe each type of leukocyte and explain their function:

A

Neutrophils are the most abundant in the body and are an early responder in innate immunity and use phagocytosis to kill microbes. Eosinophils are active in parasitic infections and allergic responses. Basophils release histamine and proteolytic enzymes. Monocytes are the largest in size and are released from the bone marrow and mature into macrophages and dendritic cells where they engage in the inflammatory response and phagocytize foreign substances and cellular debris. Macrophages have a long life span, reside in the tissues, and are the first phagocyte that organisms encounter. Neutrophils and macrophages work together on behalf of the host’s initial defense system.

How well did you know this?
1
Not at all
2
3
4
5
Perfectly
99
Q

Describe adaptive immunity:

A

Adaptive immunity, also called acquired immunity, is the second line of defense and includes both humoral and cellular mechanisms that respond to cell-specific substances known as antigens. Adaptive immunity is acquired through previous exposure to infections and other foreign agents. It can not only distinguish self from nonself but can recognize and destroy specific foreign agents based on their different antigenic properties. This response takes more time but is extremely effective.

How well did you know this?
1
Not at all
2
3
4
5
Perfectly
100
Q

What are antigens?

A

Antigens are foreign substances that elicit an adaptive immune response in the body. Adaptive immunity serves to distinguish self from nonself and to destroy specific foreign agents based on their different antigenic properties.

How well did you know this?
1
Not at all
2
3
4
5
Perfectly
101
Q

What are the primary cells of adaptive immunity?

A

The primary cells of the adaptive immune system are the lymphocytes, APCs (antigen presenting cells), and effector cells.

How well did you know this?
1
Not at all
2
3
4
5
Perfectly
102
Q

Describe B & T lymphocytes and discuss their role in immunity:

A

B lymphocytes produce the antibodies (humoral immunity) and T lymphocytes provide the cell-mediated immunity. B and T lymphocytes have the unique function as the only cells to recognize specific antigens present on the surface of pathogens and to remember them in the future.

How well did you know this?
1
Not at all
2
3
4
5
Perfectly
103
Q

What are the differences between innate and adaptive immunity (table 3.1)

A

Innate - immediate response; general patterns on microbe recognition; similar with each exposure; defense is the epithelium (skin, mucous membranes), phagocytes, inflammation, fever; includes phagocytes (monocytes/macrophages, neutrophils), NK cells, DCs; molecular components include cytokines, complement proteins, acute-phase proteins, soluble mediators.

Adaptive - dependent on exposure; large diversity and specific for each unique antigen; immunologic memory and more rapid and efficient with subsequent exposure; defenses include cell killing via tagging of antigen by antibody for removal; cellular components include T & B lymphocytes, macrophages, DCs, NK cells; molecular components include antibodies, cytokines, complement system.

How well did you know this?
1
Not at all
2
3
4
5
Perfectly
104
Q

What is the function of cellular immunity?

A

T lymphocytes make up the cellular immunity and function to activate other T and B cells, control intracellular viral infections, reject foreign tissue grafts, activate autoimmune processes, and activate delayed hypersensitivity reactions.

Cellular immunity involves activation of specific T-lymphocytes which defend the body against intracellular microbes, like viruses.

CD4+ Helper T Cells are activated and secrete cytokines, these activate many inflammatory cells (macrophages, basophils, neutrophils) which further propagates processes for pathogen destruction.

How well did you know this?
1
Not at all
2
3
4
5
Perfectly
105
Q

What are the 5 classes of immunoglobulins

A

IgG - 75%; antiviral, antibacterial, antitoxin. Crosses placenta to give immunity to fetus.
IgA - saliva, tears, colostrum, bronchial, GI, prostatic, and vaginal secretions. Local immunity on mucosal surfaces. Prevents attachment of viruses and bacteria onto epithelial cells.
IgM - lysis of microorganisms. 1st antibody produced by the fetus and immature B lymphocytes and first Ig to be produced in response to an infection. Monitoring IgM levels is helpful clinically, as increased levels indicate an active infection.
IgD - low levels; unknown functions.
IgE - least common, responsible in inflammation and allergic reactions as well as combating parasitic infections.

How well did you know this?
1
Not at all
2
3
4
5
Perfectly
106
Q

Describe the differences between active and passive immunity?

A

Active immunity is acquired from an immune response either via vaccination or from environmental exposure. Active immunity is long lasting but takes days to weeks after the first exposure to fully develop a response.

Passive immunity is immunity transferred from another source. The most common example is from mother to fetus, where the IgG antibodies are passed on either via the placenta or breast milk or colostrum. Passive immunity is short-term protection lasting only weeks to months.

How well did you know this?
1
Not at all
2
3
4
5
Perfectly
107
Q

What are type I hypersensitivity reactions?

A

Type I hypersensitivity reactions are IgE mediated reactions that develop quickly upon exposure to an antigen. Allergies fall into this category.

How well did you know this?
1
Not at all
2
3
4
5
Perfectly
108
Q

What cells are involved in type I reactions? (Know their roles.)

A

Mast cells, basophils, and eosinophils.

Mast cells and basophils release histamine, a vasodilator that relaxes vascular smooth muscle, increases permeability of capillaries and venules, and causes smooth muscle contraction and bronchial constriction.

Eosinophils are drawn in by cytokines to the site of allergen exposure

How well did you know this?
1
Not at all
2
3
4
5
Perfectly
109
Q

What are type II hypersensitivity reactions?

A

Type II hypersensitivity reactions (cytotoxic) are antibody-mediated reactions where IgG or IgM antibodies are directed against target antigens on specific host cell surfaces or tissues. The antigens may be intrinsic or extrinsic. Type II reactions occur in 4 ways:

1) Complement activated cell destruction: lysis of the cell and activation of macrophages, which then further destroys the target cells by phagocytosis.
2) Antibody dependent cell cytotoxicity: components of the innate and adaptive immune responses destroy target cells. Antibody binding on the target cell surface causes the release of chemotactic substances and teh destruction of the target cell.
4) Complement and antibody mediated inflammation - can occur rather than phagocytosis or cell lysis, when antibodies are deposited on extracellular tissue components. The activated leukocytes release injurious substances that cause inflammation and tissue damage.
5) Antibody mediated cellular dysfunction - antibody binding to specific target cell receptors does not lead to death, but rather to unwanted changes in cell function.

How well did you know this?
1
Not at all
2
3
4
5
Perfectly
110
Q

What are autoimmune disorders?

A

Autoimmune diseases occur when the body’s immune system fails to differentiate self-antigens from nonself antigens and mounts an immunologic response against host tissues.

How well did you know this?
1
Not at all
2
3
4
5
Perfectly
111
Q

What are the different types of type I hypersensitivity reactions (allergic rhinitis, anaphylaxis) and what are their appropriate treatments

A

Type I hypersensitivity reactions are IgE-mediated reactions that develop quickly upon exposure to an antigen. As this is the classic allergic response, the antigens are known as allergens. Common allergens include pollen proteins, foods, house dust mites, animal dander, household chemicals, and even pharmaceutical agents, like penicillin. Exposure can be through inhalation, ingestion, injection, or skin contact and can cause a localized or systemic reaction. Type I reactions include seasonal rhinitis, asthma, and in severe cases, anaphylaxis.

Antihistamines and intranasal corticosteroids. Allegra, Zyrtec, Benadryl. Flonase, Nasacort. Epipen.

How well did you know this?
1
Not at all
2
3
4
5
Perfectly
112
Q

What is autoreactivity?

A

If a self reactive cell is not eliminated, it can recognize the body’s own tissues and create an immune response against the self antigen. It results in the destruction of cells and tissues in the immune system.

How well did you know this?
1
Not at all
2
3
4
5
Perfectly
113
Q

What is anergy?

A

When the loss of lymphocyte response to an antigen occurs and causes a lack of cellular and/or humoral immunologic response, this is called anergy. Anergy is the state of immunologic tolerance to specific antigens.

How well did you know this?
1
Not at all
2
3
4
5
Perfectly
114
Q

What are autoantibodies?

A

Autoantibodies are produced when the immune system loses its ability to recognize self, and they act against the host’s tissues.

How well did you know this?
1
Not at all
2
3
4
5
Perfectly
115
Q

What are positive and negative selection?

A

Positive selection takes place in the thymus during maturation when T cells encounter self-peptides bound to MHC molecules. The T cells taht display no reactivity to the host’s MHC antigens are allowed to mature (positive selection).

The T cells that have a high affinity to self antigens are sorted out and undergo apoptosis (negative selection),

How well did you know this?
1
Not at all
2
3
4
5
Perfectly
116
Q

What are the 2 general causes of autoimmune diseases?

A

Genetic and environmental factors. There are susceptibility genes that combine with a trigger event.

How well did you know this?
1
Not at all
2
3
4
5
Perfectly
117
Q

How do you diagnose an autoimmune disease?

A

Diagnosis is made by history, physical, and serological findings. The following criteria must be met: evidence of an autoimmune reaction, the immunologic findings are not secondary to another condition, and no other identifiable causes are found.

How well did you know this?
1
Not at all
2
3
4
5
Perfectly
118
Q

What is Graves’ disease?

A

Graves disease is an autoimmune disorder characterized by abnormal stimulation of the thyroid gland by thyroid stimulating hormone receptor antibodies that act through the normal receptors. It is a state of hyperthyroidism, goiter, and ophthalmopathy.

How well did you know this?
1
Not at all
2
3
4
5
Perfectly
119
Q

What is SLE?

A

SLE is systemic lupus erythematosus, a chronic inflammatory disease that can affect almost any organ system. It is characterized by the presence of autoantibodies and non-self antigens.

How well did you know this?
1
Not at all
2
3
4
5
Perfectly
120
Q

What is HIV?

A

HIV is a retrovirus that selectively attacks the CD4+ T lymphocytes, the immune cells responsible for coordinating the immune response to infection. Because of this, people with HIV infection are more susceptible to severe infections with ordinarily harmless organisms.

How well did you know this?
1
Not at all
2
3
4
5
Perfectly
121
Q

How is HIV spread?

A

HIV is transmitted from one person to another through sexual contact, blood-to-blood contact, or from mother to child during pregnancy, birth, or breastfeeding. HIV is not spread via saliva or causal contact.

How well did you know this?
1
Not at all
2
3
4
5
Perfectly
122
Q

What is seroconversion?

A

Seroconversion is the term when an infected person’s blood converts from being negative for HIV antibodies to being positive.

How well did you know this?
1
Not at all
2
3
4
5
Perfectly
123
Q

What is the window period?

A

The window period is the time from infection to seroconversion, usually 1-3 months.

How well did you know this?
1
Not at all
2
3
4
5
Perfectly
124
Q

What is the latent and last phase of HIV infection?

A

The latent phase is characterized by no signs or symptoms of illness. The last phase, or AIDS illness, occurs when the CD4+ cell count falls to less than 200 cells/µL or exhibits an AIDS-defining illness.

How well did you know this?
1
Not at all
2
3
4
5
Perfectly
125
Q

What are opportunistic infections?

A

Opportunistic infections are those common organisms that do not produce infection without impaired immune function.

How well did you know this?
1
Not at all
2
3
4
5
Perfectly
126
Q

Describe the different diagnostic methods for HIV.

A

The best diagnostic method to detect HIV is the HIV antibody test, known as the enzyme immunoassay (EIA), or ELISA. If positive, it is followed by the confirmatory test, the Western blot assay. The EIA detects antibodies produced in response to HIV infection. The polymerase chain reaction (PCR) test is a nucleic acid test that can detect HIV DNA. It tests for the presence of the virus, rather than the antibody, which is helpful in diagnosing HIV infection in infants born to infected mothers. These infants would have their mother’s antibodies whether or not they have been infected.

How well did you know this?
1
Not at all
2
3
4
5
Perfectly
127
Q

What are the signs and symptoms of acute (primary phase) HIV infection (chart 3.3

A

Fever, fatigue, rash, headache, lymphadenopathy, pharyngitis, athralgia, myalgia, night sweats, GI problems, aseptic meningitis, oral or genital ulcers

How well did you know this?
1
Not at all
2
3
4
5
Perfectly
128
Q

What are the 2 phases of acute inflammation?

A

Acute inflammation can be divided into the vascular stage and the cellular stage. The vascular phase begins with temporary vasoconstriction, followed by vasodilation as a result of histamine and nitric oxide. Heat and redness appear. This leads to increased vascular permeability and exudate moves into the extravascular space, which causes an interstitial osmotic pressure increase so an increase in fluid. This leads to swelling, pain, and impaired function. As fluid movement slows, blood clotting occurs due to increased red cells, leukocytes, platelets, and clotting facto, and it helps to contain the spread of infectious microorganisms.

The cellular stage delivers leukocytes, predominantly neutrophils, to the site of injury to initiate their immune response.

How well did you know this?
1
Not at all
2
3
4
5
Perfectly
129
Q

What are the steps in the cellular phase?

A

Adhesion and margination, transmigration, and chemotaxes

How well did you know this?
1
Not at all
2
3
4
5
Perfectly
130
Q

Describe the process of chronic inflammation and discuss how it often develops.

A

Chronic inflammation may result from recurrent or progressive acute inflammatory processes. It involves macrophages and lymphocytes instead of neutrophils.

Nonspecific chronic inflammation includes an accumulation of macrophages and lymphocytes at the site of injury - they accumulate due to ongoing chemotaxis and this leads to fibroblast proliferation and scar formation.

Granulomatous lesion consists of macrophages encompassed by lymphocytes and is often associated with foreign bodies such as splinters, asbestos, sutures, and silica.

How well did you know this?
1
Not at all
2
3
4
5
Perfectly
131
Q

What are the 3 stages of wound repair?

A

There are three general stages of wound healing: (1) the inflammatory phase, (2) the proliferative phase, and (3) the wound contraction and remodeling phase.

How well did you know this?
1
Not at all
2
3
4
5
Perfectly
132
Q

What factors can inhibit wound repair?

A

Factors that affect wound healing negatively include malnutrition, decreased blood flow and oxygen delivery, impaired inflammatory and immune responses, infection, wound separation, foreign bodies, and age.

How well did you know this?
1
Not at all
2
3
4
5
Perfectly
133
Q

Explain the challenges of diagnosing autoimmune disorders.

A

There are over 80 identified, many with overlapping presentations. Many manifestations are nonspecific and are seen in other non-autoimmune diseases. Blood testing isn’t perfect either, as some tests are more generic and can be elevated in the presence of other diseases.

How well did you know this?
1
Not at all
2
3
4
5
Perfectly
134
Q

Explain the challenges of diagnosing autoimmune disorders.

A

There are over 80 identified, many with overlapping presentations. Many manifestations are nonspecific and are seen in other non-autoimmune diseases. Blood testing isn’t perfect either, as some tests are more generic and can be elevated in the presence of other diseases.

How well did you know this?
1
Not at all
2
3
4
5
Perfectly
135
Q

What allows the lymphocyte to differentiate between self and foreign moelcules

A

Major histocompatibility complex (MHC) molecule

How well did you know this?
1
Not at all
2
3
4
5
Perfectly
136
Q

Which immunoglobulin passes immune factors from the mother to the fetus

A

IgG

How well did you know this?
1
Not at all
2
3
4
5
Perfectly
137
Q

A 12-year-old female presents with itchy eyes, nasal congestion and drainage, and sneezing every spring when the pollen count is high. (1) Explain the immunologic mechanisms that are responsible for her symptoms. (2) What type(s) of treatment might be used to relieve her symptoms?

A

The 12 year old is experiencing a type I hypersensitivity. When allergens like pollen are detected in the body, IgE is activated and triggers an immunoresponse, including the release of histamine in the body. This is because pollen, and other allergens, are antigens and known to the body as “non-self.”

Two treatments that could help relieve her sypmtoms include an antihistamine and/or corticosteroid. A second generation drug such as Claritin or Zyrect, combined with a nasal spray like Flonase should help relieve her symptoms.

(1) Mast cells, basophils, and eosinophils play an important role in the development of type I reactions because they contain the chemical mediator histamine. A primary or initial-phase response is vasodilation, vascular leakage, and smooth muscle contraction. A secondary or late-phase response is characterized by more intense infiltration of tissues with eosinophils and other acute and chronic inflammatory cells, as well as tissue damage. (2) Antihistamines and intranasal corticosteroids are the mainstay of treatment.

How well did you know this?
1
Not at all
2
3
4
5
Perfectly
138
Q

Can an antibiotic alone heal an abscess?

A

Antibiotics are unable to breakthrough the protective layer of fibers formed around the abscess. In order to heal the abscess it must be lanced and drained.

How well did you know this?
1
Not at all
2
3
4
5
Perfectly
139
Q

An appendectomy is performed on a 16-year-old boy who is hospitalized for right lower quadrant abdominal pain of 18 hours’ duration. The surgical specimen is edematous and erythematous. An infiltrate of which cells would be most characteristic of the process occurring here?

A

Neutrophils

How well did you know this?
1
Not at all
2
3
4
5
Perfectly
140
Q

Which hypersensitivity reactions can be treated with the administration of epinephrine?

A

Type I

How well did you know this?
1
Not at all
2
3
4
5
Perfectly
141
Q

A 32-year-old woman presents to the clinic for her initial obstetrics visit, about 10 weeks into her pregnancy. She is in a monogamous relationship. (1) Should an HIV test be part of her initial blood work? Why?

A

Yes. Transmission from mother to infant is the most common way that children become infected. The CDC recommends all people between ages 13 and 64 be routinely screened for HIV. Earlier detection and treatment lead to better outcomes.

How well did you know this?
1
Not at all
2
3
4
5
Perfectly
142
Q

An infant is born, and its initial antibody test is positive for HIV. (2) Does this mean the infant is infected? What would be the more appropriate test to detect HIV in this infant?

A

No. The polymerase chain reaction (PCR) test is a nucleic acid test that can detect HIV DNA. It tests for the presence of the virus, rather than the antibody, which is helpful in diagnosing HIV infection in infants born to infected mothers. These infants would have their mother’s antibodies whether or not they have been infected.

How well did you know this?
1
Not at all
2
3
4
5
Perfectly
143
Q

The cells primarily responsible for the tissue building that occurs during the proliferative phase of wound healing are known as what?

A

Fibroblasts

How well did you know this?
1
Not at all
2
3
4
5
Perfectly
144
Q

What cell mediator is the primary cause of vasodilation seen in acute inflammation?

A

Histamine

How well did you know this?
1
Not at all
2
3
4
5
Perfectly
145
Q

Which lab value will typically be increased in a viral infection?

A

Lymphocytes

How well did you know this?
1
Not at all
2
3
4
5
Perfectly
146
Q

Which lab value will typically be increased in a viral infection?

A

Lymphocytes

How well did you know this?
1
Not at all
2
3
4
5
Perfectly
147
Q

What is the function of blood?

A

The function of blood is to transport gases, nutrients, wastes, and hormones. It helps to regulate temperature, pH, and fluid volume.

How well did you know this?
1
Not at all
2
3
4
5
Perfectly
148
Q

What are the three stages of hemostasis?

A

Hemostasis is divided into vascular constriction, formation of the platelet plug, and blood coagulation in order to stop the flow of blood.

How well did you know this?
1
Not at all
2
3
4
5
Perfectly
149
Q

What is vitamin K needed to synthesize?

A

Vitamin K is needed to synthesize protein coagulation factors II, IX, and X, prothrombin, and protein C. A deficiency of vitamin K or liver failure will result in abnormal bleeding as prothrombin will not be made.

How well did you know this?
1
Not at all
2
3
4
5
Perfectly
150
Q

Discuss the intrinsic and extrinsic pathways:

A

The intrinsic and extrinsic pathways both form prothrombin activator. The intrinsic pathway begins with activation of factor XII and can cause clotting in 1 to 6 minutes, considered the slower process. The extrinsic pathway can cause clotting in 15 seconds and begins with the release of tissue thromboplastin from the subendothelial cells. Both pathways converge to activate factor X, the conversion of prothrombin to thrombin. Thrombin then converts fibrinogen to fibrin, the material needed to stabilize a clot. Both methods are needed for successful hemostasis.

How well did you know this?
1
Not at all
2
3
4
5
Perfectly
151
Q

What are the natural and drug therapy anticoagulants?

A

The natural anticoagulants found in the body are antithrombin III, protein C, and protein S. The drug therapy anticoagulants are Warfarin (inhibits coagulation factors dependent on vitamin K for synthesis), Heparain (stops the formation of fibrin), Dabigatran (oral anticoagulant).

How well did you know this?
1
Not at all
2
3
4
5
Perfectly
152
Q

What is peripheral vascular resistance?

A

Peripheral vascular resistance is the collective resistance of all the vessels in the peripheral circulation. Resistance is the opposition to flow caused by friction between the moving blood and the stationary vessel wall.

How well did you know this?
1
Not at all
2
3
4
5
Perfectly
153
Q

Hemodynamics or blood flow is affected by what factors?

A

Hemodynamics are affected by blood pressure, vessel resistance, and blood flow. Resistance is described above. Blood flow is affected by the blood vessel radius and blood viscosity.

How well did you know this?
1
Not at all
2
3
4
5
Perfectly
154
Q

What is cardiac output?

A

Cardiac output is the volume of blood the heart pumps per minute. It can be calculated by multiplying the heart rate by the stroke volume, which is the blood pumped out by the ventricle with each beat. CO = SV x HR.

How well did you know this?
1
Not at all
2
3
4
5
Perfectly
155
Q

What is preload?

A

Preload is the end-diastolic pressure when the ventricle has been filled and represents the volume work of the heart. It’s called preload because it is the work on the heart before the contraction begins. It is mainly the result of venous return of blood to the heart and the stretch of the cardiac muscle fibers. In short, it is the volume of blood in the ventricles the heart has to eject.

How well did you know this?
1
Not at all
2
3
4
5
Perfectly
156
Q

What is afterload?

A

Afterload is the pressure the heart exerts to overcome the pressure in the heart vessels to be able to eject the blood from the heart.

How well did you know this?
1
Not at all
2
3
4
5
Perfectly
157
Q

What is cardiac contractility?

A

Cardiac contractility is the ability of the heart to change its force of contraction without changing its resting (diastolic) length. This is strongly influenced by the number of calcium ions available to participate int he contractile process. Contractility increases cardiac output independent of preload or afterload.

How well did you know this?
1
Not at all
2
3
4
5
Perfectly
158
Q

What is an inotrope?

A

An inotrope is an effect that increases or decreases the contractile force. Positive inotropic effects (increasing contractile force) include the sympathetic nervous system, afterload, catecholamines, heart rate, drugs, intracellular calcium levels. Negative inotropic effects (decrease contractile force) include the parasympathetic nervous system, heart failure, hypoxia, and drugs.

How well did you know this?
1
Not at all
2
3
4
5
Perfectly
159
Q

What is mean arterial pressure?

A

Mean arterial pressure is the average pressure in the arterial pressure in the arterial system during a cardiac cycle and is a good indicator of tissue perfusion. The mean arterial pressure is the product of the cardiac output (SVHR) and the peripheral vascular resistance (MPA = COPVR)

How well did you know this?
1
Not at all
2
3
4
5
Perfectly
160
Q

Understand the renin-angiotensin-aldosterone system

A

In the RAAS, kidneys secrete renin, which gets converted eventually into angiotensin II, that forms ADH and aldosterone to reabsorb sodium and water. It increases fluid retention and vasoconstriction. This worsens both pre and afterload. RAAS plays a vital role in blood pressure regulation. If there are changes in extracellular fluid, extracellular sodium levels, a drop in blood pressure, or increased sympathetic activity, the kidney will release renin. In the bloodstream, renin converts angiotensinogen into angiotensin I. Angiotensin I is converted to angiotensin II in the lungs by ACE (angiotensin-converting enzyme). Angiotensin II is a vasoconstrictor, mainly of the arterioles. Arteriole constriction will increase the PVR, regulating blood pressure for a short period of time. Aldosterone will increase salt and water retention by the kidney contributing to the long-term regulation of blood pressure.

How well did you know this?
1
Not at all
2
3
4
5
Perfectly
161
Q

Be familiar with hypercoagulability states

A

Hypercoagulability is the accelerated activity of the clotting system. There are conditions that create increased platelet activity and conditions that cause increased activity of the coagulation pathways. Arterial thrombi are often a result of conditions characterized by an increase in platelet numbers or restricted blood flow with platelet adhesion. Venous thrombi are associated with conditions, genetic in nature or acquired, that decrease the amount of anticoagulation factors or produce a stasis of blood, which in turn increases the number of pro-coagulation factors.

How well did you know this?
1
Not at all
2
3
4
5
Perfectly
162
Q

What is thrombocytosis?

A

Thrombocytosis is defined as a platelet count greater than 450,000/microL. The overall platelet count is regulated by a negative feedback loop. If the platelet count falls below normal limits, hormonal thrombopoietin levels will rise to increase the number of platelets, and when the count reaches normal limits, thrombopoietin levels will decrease. Primary thrombocytosis is due to a disorder of the hematopoietic stem cells within the bone marrow resulting in abnormally high levels of thrombopoietin, which leads to an overproduction of platelets. Secondary thrombocytosis is is also an abnormal thrombopoietin production which overproduces platelet production, but is due to a disease state (tissue damage from surgery, infection, cancer, and chronic inflammatory conditions).

How well did you know this?
1
Not at all
2
3
4
5
Perfectly
163
Q

What are the most common causes of inherited disorders of hypercoagulability?

A

Inherited disorders of hypercoagulability are most commonly caused by a mutation of the factor V gene or prothrombin gene. The mutant factor V cannot be inactivated by protein C which helps to regulate anticoagulation. As a result, a factor V mutation will lead to an increased risk fro developing deep vein thrombosis.

How well did you know this?
1
Not at all
2
3
4
5
Perfectly
164
Q

What are acquired or secondary factors of hypercoagulability that lead to venous stasis?

A

Secondary hypercoagulability that leads to venous stasis can be caused by bed rest, use of oral contraceptives, smoking, pregnancy and postpartum, sepsis, heart failure, cancer.

How well did you know this?
1
Not at all
2
3
4
5
Perfectly
165
Q

What is thrombocytopenia?

A

Thrombocytopenia is a reduction in platelets which increase a risk of bleeding.

How well did you know this?
1
Not at all
2
3
4
5
Perfectly
166
Q

Be familiar with hemophilia A and DIC pathology, clinical presentation, diagnosis, and treatment.

A

Hemophilia A is a recessive genetic disorder that causes an insufficient production in factor VIII which promotes clotting and adhesion of platelets to the vessel wall, bleeding in the soft tissues, GI tract, and joints. Trauma prevention is very important, Factor VIII replacement therapy is the current treatment. Disseminated intravascular coagulation (DIC) is characterized by widespread coagulation and bleeding. DIC begins with the activation of the extrinsic or intrinsic coagulation cascade, the result of unregulated generation of thrombin, resulting in systemic formation of fibrin. All the major anticoagulant levels are decreased. Microthrombi are formed causing vessel occlusion and tissue ischemia. As all availability coagulation proteins and platelets are being used to form clots, severe hemorrhage ensues. Treatment is focused on managing the primary disease, replacing clotting components, and preventing further activation of clotting mechanisms.

How well did you know this?
1
Not at all
2
3
4
5
Perfectly
167
Q

Be familiar with hemophilia A and DIC pathology, clinical presentation, diagnosis, and treatment.

A

Hemophilia A is a recessive genetic disorder that causes an insufficient production in factor VIII which promotes clotting and adhesion of platelets to the vessel wall, bleeding in the soft tissues, GI tract, and joints. Trauma prevention is very important, Factor VIII replacement therapy is the current treatment. Disseminated intravascular coagulation (DIC) is characterized by widespread coagulation and bleeding. DIC begins with the activation of the extrinsic or intrinsic coagulation cascade, the result of unregulated generation of thrombin, resulting in systemic formation of fibrin. All the major anticoagulant levels are decreased. Microthrombi are formed causing vessel occlusion and tissue ischemia. As all availability coagulation proteins and platelets are being used to form clots, severe hemorrhage ensues. Treatment is focused on managing the primary disease, replacing clotting components, and preventing further activation of clotting mechanisms.

How well did you know this?
1
Not at all
2
3
4
5
Perfectly
168
Q

How can anemias be classified according to size and color?

A

Anemia can be characterized as normochromic (normal color) or hypochromic (decreased color). Red cells can be classified as normocytic (normal size), microcytic (small cells), and macrocytic (large cells).

How well did you know this?
1
Not at all
2
3
4
5
Perfectly
169
Q

Be familiar with sickle cell disease, thalassemia, iron deficiency anemia, vitamin B12 and folate deficiency anemia – pathology, clinical presentation, diagnosis, and treatment.

A

Sickle cell disease is a genetic disorder characterized by chronic hemolytic anemia, pain, and organ failure caused by abnormal hemoglobin S. Sickle cell disease is a single mutation in the hemoglobin molecule. In conditions of decreased oxygen levels, the cell becomes sickled as the abnormal hemoglobin sticks together. With proper oxygenation, the cells may return to normal shape; with, the cell will remain permanently sickled. This may lead to hemolysis or occlusion. Clinical presentation can include leg ulcers, damage to many organs, acute chest syndrome, asplenia. Diagnosis is confirmed by screening at birth, hemoglobin electrophoresis. Treatment is focused on preventing sickling episodes like infections, cold exposure, severe physical exertion, acidosis, and dehydration.

Iron Deficiency Anemia is the loss of iron through bleeding or increased demands. Iron is a component of heme and decreased levels lead to impaired oxygen delivery. Clinical presentation includes fatigue, dyspnea, angina, and tachycardia, pica. IDA presents with low hemoglobin and hematocrit, decreased iron stores, low serum iron and ferritin levels. Prevention is key - eating a diet rich in iron containing foods, taking iron fortified vitamins, and treating any underlying chronic blood loss.

Thalassemia is a genetic disease with defective hemoglobin synthesis and hemolysis. It is a hypochromic, microcytic anemia due to decreased synthesis of the affected chain coupled with continued production and accumulation of the unaffected globin chain. People with Thalassemia major must undergo blood transfusions in life or risk severe growth retardation. The ineffective hematopoiesis and hemolysis leads to increased erythropoietin secretion and hyperplasia in the bone marrow which impairs bone growth and leads to osteoporosis or osteopenia.

Vitamin B12 Deficiency Anemia is caused by impaired DNA synthesis that results in enlarged red blood cells. B12 is necessary for DNA synthesis and development of the cel nuclei. The clinical presentation includes moderate to severe anemia and mild jaundice. The the MCV is elevated due to large cells and the MCHC is normal. There can be neurologic changes as well caused by meylin breakdown. Diagnosed by low B12 serum level. Treatment includes lifelong intramuscular injections or high oral doses.

Folic Acid Deficiency is similar to B12 deficiency, as it is required for DNA synthesis and red blood cell maturation. Most common causes are malnutrition. Supplementation should support people with this condition.

How well did you know this?
1
Not at all
2
3
4
5
Perfectly
170
Q

What is neutropenia and what are people who have it at risk for?

A

Neutropenia is the abnormal decrease in the circulating neutrophils, thus weakening the immune system. It is the result of decreased production, increased destruction, or relocation to tissue from the bloodstream. For some it is congenital, for others it can be caused by lupus, rheumatoid arthritis, HIV, and hematologic cancers.

How well did you know this?
1
Not at all
2
3
4
5
Perfectly
171
Q

What are the five types of lipoproteins? Which one is the good and bad cholesterol?

A

High Density Lipoprotein is the good cholesterol. Low density lipoprotein is the bad cholesterol. There is also very low density lipoprotein, intermediate density lipoprotein, and chylomicrons.

How well did you know this?
1
Not at all
2
3
4
5
Perfectly
172
Q

Where does lipoprotein synthesis take place?

A

Lipoprotein synthesis takes place in the liver and the small intestine.

How well did you know this?
1
Not at all
2
3
4
5
Perfectly
173
Q

What are xanthomas?

A

Xanthomas are cholesterol deposits that will develop along the tendons with hypercholesterolemia. This can lead to atherosclerosis.

How well did you know this?
1
Not at all
2
3
4
5
Perfectly
174
Q

Be familiar with hyperlipidemia pathology, clinical presentation, diagnosis, and treatment.

A

Hyperlipidemia is the the excess of lipids in the blood. Nutrition, genetics, medications, comorbid conditions, and metabolic disease can all raise blood lipid levels. Treatment focuses on dietary and therapeutic lifestyle changes like increasing physical activity, weight reduction, and smoking cessation. If those don’t work, hyperlipidemia drugs can work.

How well did you know this?
1
Not at all
2
3
4
5
Perfectly
175
Q

What is atherosclerosis and what is it a major risk factor of?

A

Atherosclerosis is the hardening of the arteries and it is a major risk for ischemic heart disease, stroke, and peripheral vascular disease. Hyperlipidemia.

How well did you know this?
1
Not at all
2
3
4
5
Perfectly
176
Q

Be familiar with atherosclerosis pathology and clinical presentation.

A

The etiology is the “response to injury theory” in which injury of a blood vessel (due to inflammation, immune mechanisms, high cholesterol, hypertension, smoking) results in macrophages scavenging the excess cholesterol. They oxidize the LDL and become foam cells which decrease cholesterol from circulation but will release factors that will injure the endothelial cells and the blood vessel. Foam cells will keep collecting and form a lesion on the blood vessel wall, the macrophages will secrete growth factors that secrete cell proliferation. The macrophages will die and leave behind their necrotic debris and then we start to accumulate lipids and this will generate our plaque. The lesion will be covered by smooth muscle, extracellular matrix, and will inhibit flow and increase blood pressure and resistance. If the lesion ruptures, it can cause hemorrhage or a thrombosis. This can present as angina, impotence, wounds that wont heal.

How well did you know this?
1
Not at all
2
3
4
5
Perfectly
177
Q

What causes coronary artery disease (CAD)?

A

Coronary artery disease (CAD) is impaired coronary blood flow that causes heart disease. CAD leads to myocardial ischemia and angina, myocardial infarction (heart attack), cardiac arrhythmias, abnormalities in conduction, heart failure, and even death.

How well did you know this?
1
Not at all
2
3
4
5
Perfectly
178
Q

What are the major risk factors of CAD?

A

The major risk factors of CAD are smoking, chronic high blood pressure, increased LDL, decreased HDL, diabetes, age, abdominal obesity, and decreased physical activity.

How well did you know this?
1
Not at all
2
3
4
5
Perfectly
179
Q

Be familiar with the methods used to assess coronary blood flow and myocardial perfusion (electrocardiography, exercise stress testing, echocardiography, Doppler ultrasonographic imaging, cardiac MRI and CT, and cardiac catherization and angiography).

A

Electrocardiography - records electrical potential differences during the cardiac cycle. It is most frequently used cardiovascular procedure that can diagnose and identify ventricular conduction defects, arrhythmias, electrolyte imbalances, drug effects, and genetically mediated electrical or structural abnormalities.

Exercise stress test - observes cardiac function under stress (running on a treadmill). Used for adults with suspected ischemic heart disease, but also physiologic response preoperatively, post MI or revascularization, or the severity of arrhythmias.

Echocardiography - best method to check for structure and function of the heart.

Doppler Ultrasonographic imaging

Cardiac MRI and CT

Cardiac Catheterization - a catheter is inserted into the great vessels and chambers of the heart to inject radiographic contrast.

Angiography

How well did you know this?
1
Not at all
2
3
4
5
Perfectly
180
Q

Be familiar with the disease states in acute coronary syndrome.

A

Acute coronary syndrome (ACS) represents a range of acute ischemic heart diseases ranging from unstable angina to myocardial infarction. It results from disruption of an atherosclerotic plaque.

How well did you know this?
1
Not at all
2
3
4
5
Perfectly
181
Q

What are the 2 different ways heart failure can be defined?

A

Heart failure can be described in terms of systolic dysfunction (the ejection of blood during systole is impaired) or diastolic dysfunction (impaired filling capability of the heart during diastole).

How well did you know this?
1
Not at all
2
3
4
5
Perfectly
182
Q

Be familiar with heart failure pathology, clinical presentation, diagnosis, and treatment.

A

Clinical manifestations of heart failure include shortness of breath, fatigue, weakness, fluid retention. Diagnosis depends on history, physical exam, lab studies, electrocardiography, etc. Treatment is aimed at stabilizing and correcting the cause of the cardiac dysfunction to receive symptoms, improve quality of life, and reduce risk factors. Diuretics, ACE inhibitors or angiotensin II receptor blockers, beta blockers, digoxin, and vasodilators can be used.

How well did you know this?
1
Not at all
2
3
4
5
Perfectly
183
Q

Be familiar with the different types of cardiomyopathies.

A

Cardiomyopathy is a disease of the heart muscle that causes it to be enlarged, thick, or rigid. Hypertrophic cardiomyopathy is the thickening of the heart wall, especially in the left ventricle. It makes the chamber smaller which causes an abnormal ventricular filling and decreased stroke volume. Dilated cardiomyopathy causes the left ventricle to be enlarged and cannot contract properly to conduct the blood. Restrictive myopathy causes the heart to lose elasticity so it does not expand well to fill with blood.

How well did you know this?
1
Not at all
2
3
4
5
Perfectly
184
Q

Be familiar with the different types of arrhythmias

A

Cardiac arrhythmias are electrical impulses in the cardiac conduction system that fail to coordinate causing the heart rate to increase, decrease, or have an irregular rhythm. There are Tachyarrhythmias in which the heart rate is above 100 beats/min which decreases the fill time of the heart and decreases the CO. Bradyarrhthmias slow the heart rate to less than 60 beats/min and can decrease profusion of our organs.

How well did you know this?
1
Not at all
2
3
4
5
Perfectly
185
Q

What are the current stages of hypertension?

A

Currently, normal blood pressure is defined as less than 120/80. Elevated blood pressure is < 130/80; hypertension stage 1 is < 140/90; and hypertension stage 2 is > 140/90.

How well did you know this?
1
Not at all
2
3
4
5
Perfectly
186
Q

Be familiar with hypertension pathology, clinical presentation, diagnosis, and treatment.

A

There are both non-modifiable risk factors (age, family history, gender, race) and modifiable (salt, excessive calorie, alcohol intake). Hypertension can promote and accelerate plaque formation and rupture, it is also a risk factor for CAD, peripheral artery disease, stroke, and heart failure. Prolonged elevation of blood pressure places an increased workload on the left ventricle as it increases the pressure the heart must pump against as it ejects blood into the systemic circulation. Over time, the left ventricle hypertrophies as it compensates for the increased pressure. Left ventricular hypertrophy is a major risk factor for heart disease and even death. Hypertension is diagnosed with repeated blood pressure measurement. Treatment is usually lifestyle changes as well as medication like diuretics, ACE inhibitors, calcium channel receptor blockers.

How well did you know this?
1
Not at all
2
3
4
5
Perfectly
187
Q

What is cardiogenic shock?

A

Cardiogenic Shock is occurs when the heart fails to pump blood sufficiently to meet the body’s needs.

How well did you know this?
1
Not at all
2
3
4
5
Perfectly
188
Q

What causes cardiogenic shock?

A

The most common cause is extensive damage to the left ventricle from a myocardial infarction where there is decreased stroke volume and cardiac output which causes low oxygen and perfusion.

Myocardial infarction, myocardial contusion, sustained arrhythmias, or cardiac surgery

How well did you know this?
1
Not at all
2
3
4
5
Perfectly
189
Q

What is the most specific test for diagnosing iron deficiency anemia

A

Ferritin

How well did you know this?
1
Not at all
2
3
4
5
Perfectly
190
Q

What can increase the change of clotting?

A

Protein C deficiency

How well did you know this?
1
Not at all
2
3
4
5
Perfectly
191
Q

What are two natural anticoagulants

A

Proteins C and S

How well did you know this?
1
Not at all
2
3
4
5
Perfectly
192
Q

Hemolytic anemias are characterized by

A

premature destruction of red blood cells, decrease in erythropoiesis, and sickle cell disease and thalassemias are included.

How well did you know this?
1
Not at all
2
3
4
5
Perfectly
193
Q

Three ways to treat or prevent a sickling crisis in sickle cell disease:

A

hydroxyurea, immunizations, treating infections

How well did you know this?
1
Not at all
2
3
4
5
Perfectly
194
Q

The spleen of a patient with sickle cell anemia would be expected to be:

A

shrunken

How well did you know this?
1
Not at all
2
3
4
5
Perfectly
195
Q

What do fatty streaks of atherosclerotic lesions consist of?

A

Macrophages and smooth muscle cells

How well did you know this?
1
Not at all
2
3
4
5
Perfectly
196
Q

What is the description and use of the various cardiac procedures: cardiac catheterization, exercise stress testing, electrocardiography, echocardiography

A

Cardiac catheterization:
A catheter is inserted into the great vessels and chambers of the heart and it is best to assess need for coronary artery bypass graft and and stenting.

Exercise stress testing:
Observes cardiac function under stress and is best to assess suspected ischemic heart disease.

Electrocardiography:
Records electrical potential differences during the cardiac cycle and is best to assess for arrhythmias and myocardial infarction.

Echocardiography:
Checks for structure and function of the heart and is best to assess ventricular function and heart failure.

How well did you know this?
1
Not at all
2
3
4
5
Perfectly
197
Q

What is the description and use of the various cardiac procedures: cardiac catheterization, exercise stress testing, electrocardiography, echocardiography

A

Cardiac catheterization:
A catheter is inserted into the great vessels and chambers of the heart and it is best to assess need for coronary artery bypass graft and and stenting.

Exercise stress testing:
Observes cardiac function under stress and is best to assess suspected ischemic heart disease.

Electrocardiography:
Records electrical potential differences during the cardiac cycle and is best to assess for arrhythmias and myocardial infarction.

Echocardiography:
Checks for structure and function of the heart and is best to assess ventricular function and heart failure.

How well did you know this?
1
Not at all
2
3
4
5
Perfectly
198
Q

Explain the differences in the pathophysiology, symptoms, and whether nitroglycerin and/or rest will relieve symptoms in stable angina, unstable angina, and myocardial infarction.

A

With atherosclerotic disease, the coronary vessel lumens are narrowed and blood supply to the heart is diminished. With exertion, the vessels cannot adequately vasodilate. This can manifest as chest pain or tightness and/or shortness of breath. Stable angina is experienced as a pain that radiates from the arm up to the neck and chest during moments of exertion. This takes place in atherosclerosis with the narrowing of the blood vessels that doesn’t allow for appropriate cardiac output even though the heart is working to compensate for the narrowed vessels. The result of the lack of needed oxygen and the heart muscle contractions is a build up of lactic acid and results in an acidic state, hence the pain. Nitroglycerin and rest should relieve some of the symptoms.

Stable angina- The heart is stable at rest but cannot keep up with the demand of exertion. Stable angina typically has a >70% stenosis of the coronary arteries. It is relieved by rest or nitroglycerin (vasodilator).

Unstable angina- With increasing occlusion of the coronary vessel(s), the demands of the resting heart may become too great. When the patient experiences angina without exertion, or when the level of exertion necessary to cause anginal symptoms decreases, this is called unstable angina. It can be relieved by nitroglycerin, but not rest. Unstable angina means one or more coronary vessels is nearly totally occluded.

Myocardial infarction- If a plaque ruptures, coronary thrombosis can cause complete occlusion. If collateral flow is inadequate, the tissue supplied becomes ischemic. Without oxygen, a portion of the muscle can die. MI can present as chest pain unrelieved by rest, sometimes radiating down the left ram or into the jaw or neck, dyspnea, nausea/vomiting, sweating, and/or other signs of distress. Other symptoms include a crushing pain or like “something is sitting on my chest.” Women may present with weakness, fatigue, shortness of breath, or GI complaints. Symptoms are not relieved by nitroglycerin.

How well did you know this?
1
Not at all
2
3
4
5
Perfectly
199
Q

Name 2 biomarkers that elevate during a myocardial infarction:

A

Troponin and creatine kinase MB (CK-MB)

200
Q

esides chest pain, name 2 other symptoms a person may experience during a myocardial infarction:

A

Pain radiating down the left ram or into the jaw or neck, dyspnea (shortness of breath), nausea/vomiting, sweating, weakness, fatigue, or GI complaints.

201
Q

The most common cause of death immediately after the onset of acute myocardial infarction is:

A

Arrhythmia

202
Q

Which arrhythmia is the most common chronic arrhythmia and incidence increases with age?

A

Atrial fibrillation

203
Q

The laboratory test that measures the volume of red cells in 100 mL of blood is _____.

A

hematocrit

204
Q

Approximately 55% of blood volume consists of _____.

A

Plasma

205
Q

Thrombocytes are also known as _______.

A

Platelets

206
Q

A 45-year-old man has a blood pressure at a screening program of 142/90. His father has a history of hypertension and heart attack. His grandfather died of a stroke. He is 20 lbs overweight, likes eating salty foods, and enjoys drinking beer. He states he is too tired and busy to exercise.

According to JNC 8 guidelines, what category does this patient’s blood pressure fall into?
What is hypertension a risk factor for?
What lifestyle modifications would you suggest for them?
Name one medication therapy for hypertension and its mechanism of action.

A

Hypertension stage 2
Atherosclerosis, CAD (MI), heart failure, stroke, peripheral artery disease
Lose weight, exercise, decrease salt intake, decrease alcohol intake
Diuretics, like thiazides, lower blood pressure by decreasing vascular volume;
ACE inhibitors inhibit the conversion of angiotensin I to angiotensin II, thereby reducing vasoconstriction, aldosterone levels;

Calcium channel receptor blockers inhibit the movement of calcium into cardiac and vascular smooth muscle. They reduce vascular smooth muscle tone (vasodilation); reduce cardiac contractility and heart rate, and decrease cardiac output

207
Q

The respiratory system can be divided into what 2 structures?

A

Conducting airways and respiratory tissues

208
Q

What are the levels of branching?

A

It goes from the primary bronchi, secondary bronchi, branching bronchioles to the terminating bronchioles, to the alveolar ducts, and alveoli.

209
Q

Where is the site of gas exchange?

A

Gas exchange takes place in the alveoli.

210
Q

What cells are in the alveolar epithelium?

A

Type 1 and II alveolar cells, macrophages

211
Q

Describe what happens during inspiration and expiration:

A

During inspiration, air from the outside travels down the pressure gradient from the high pressure area outside the body into the low pressure area down inside the lungs. In order to lower pressure in the thoracic cavity, neurosignals will cause the intercostal muscles to move the ribcage outward and the diaphragm to move downwards. As the size of the thoracic cavity increases, the internal pressure will drop, and the air will flow into the body, through the brachial tree, and into the lungs.

During expiration, we breathe out to eliminate carbon dioxide. In order to move carbon dioxide out of the body and into the air, the thoracic cavity will decrease in size by relaxing the intercostal muscles and moving the diaphragm upwards, which will increase the pressure in the thoracic cavity. Thus, the air will move out of the lungs and out of the body.

212
Q

What occurs with the diaphragm during inspiration and expiration?

A

The diaphragm is the main muscle of inspiration. When the diaphragm contracts (inspiration), the chest expands. Upon expiration, the chest cavity decreases and pressure inside increases.

213
Q

What is lung compliance? What factors affect it?

A

Lung compliance is the ability of the lungs to stretch, expanding its shape, and then return to it’s starting shape. The lung tissue has elastic fibers as part of the connective tissue to allow this expansion and recoil. If the lungs are not able to recoil, it can be difficult to exhale and if they become less compliant it can create stiffness making it difficult to stretch to accomodate air.

214
Q

What is ventilation? Perfusion? Diffusion?

A

Ventilation is the movement of gases into and out of the lungs. Perfusion is the process that allows blood flow to help facilitate gas exchange. Diffusion is the movement of gases across the alveolar-capillary membrane.

215
Q

What is the difference between a shunt and dead air space

A

A shunt is formed when blood moves from the pulmonary circulation (right side of the heart) to systemic circulation (left side of the heart) without being oxygenated. In an anatomic shunt, blood moves from the venous to the arterial side without moving through the lungs. In a physiologic shunt, blood moves through unventilated parts of the lung creating a mismatch of ventilation and perfusion.

Anatomic dead air space is the volume of air taken in that does not undergo gas exchange. This air would be found in the conducting airways to the terminal bronchioles. Alveolar dead air space are alveoli that are ventilated but not perfused (no blood flow). Physiologic dead air space is the sum of the anatomic and alveolar dead space.

216
Q

What is oxyhemoglobin?

A

Oxyhemoglobin is the term to describe when hemoglobin is bound with oxygen.

217
Q

What is affinity?

A

Affinity of the hemoglobin molecule is the degree to which it is able to bind oxygen.

Each hemoglobin molecule can bind up to 4 molecules of oxygen and with each bound oxygen moelcule, it change shape to make each consecutive oxygen molecule easier to bind. The affinity of hemoglobin for oxygen increases with hemoglobin saturation. As hemoglobin releases oxygen into the surrounding tissues, the affinity must decrease. Affinity is influenced by pH, carbon dioxide concentration, and body temperature. It binds more readily to oxygen as the blood pH increases and under conditions of decreased body temperature and CO2 concentration. Conversely, hemoglobin releases oxygen more readily in conditions of decreased pH (acidosis), increased CO2 concentration, and fever.

218
Q

How is carbon dioxide transported in the blood?

A

Carbon dioxide is transported in the blood mostly as bicarbonate, also attached to hemoglobin, and sometimes as dissolved carbon dioxide.

The amount of dissolved CO2 and the level of bicarbonate in the blood influences the acid-base balance. Once diffused into the red blood cells, it can either combine with hemoglobin or form carbonic acid (CO2 + water). This process is catalyzed in the red blood cells because of an enzyme called carbonic anhydrase.

219
Q

How is breathing controlled?

A

Breathing can be controlled by automatic regulation mechanisms by chemoreceptors and lung receptors. Chemoreceptors monitor blood levels of oxygen, carbon dioxide, and pH and adjusts ventilation rates accordingly. There are central chemoreceptors surrounded by cerebral spinal fluid in the brain that can sense changes in the blood PCO2 levels because CO2 from the blood diffuses into the CSF, freeing up hydrogen which then stimulates the chemoreceptors. And increase in PCO2 produces short term increase in ventilation. Peripheral chemoreceptors are located in the carotid and aortic bodies and they monitor arterial blood oxygen levels, however are not stimulated until PO2 levels fall below 60 mm Hg.

Lung receptors monitor breathing patterns and lung function.

Breathing can also be controlled by voluntary mechanisms giving temporary control of breathing in response to activities like speaking, singing, or holding breath.

220
Q

What are the characteristics of COPD?

A

Chronic Obstructive Pulmonary Disease (COPD) can manifest as emphysema or chronic bronchitis. It is a progressive and irreversible disease. The mechanisms that obstruct airflow include inflammation, fibrosis of the bronchial wall, increased mucus secretion, and decreased elastic fibers and alveolar tissue.

Emphysema is the enlargement of the airspaces after the terminal bronchioles because of destruction of the alveolar walls and associated capillaries. It decreases surface area for diffusion of oxygen and CO2 gases, and causes a loss of elastic recoil in the lungs.

The main causes of emphysema are smoking which triggers an inflammatory response. Neutrophils and macrophages release a proteolytic enzyme called elastase that damages and breaks down the elastic fibers in the lung tissues, which compromise the lungs recoil ability and compliance. Normally elastase is inhibited by the AAT enzyme, but smoking decreases the enzyme’s activity. The enlargement of the airspaces in the lungs means the alveoli can collapse and trap air in them, which leads to hyperinflation of the lungs - further stretching the tissue and causing a loss of elasticity. In hyperinflation there’s no more room in the alveoli to expand to take in new air and the trapped air is going to increase the CO2 concentration and lead to a lowering of the pH.

The signs and symptoms of emphysema include chronic hypoxia and hypercapnia, chronic cough, dyspnea, wheezing, barrel chest, pursed lip breathing, and respiratory distress. (Pink Puffer)

Chronic bronchitis is a productive cough that lasts for 3+ months for two or more consecutive years. It is mostly the product of smoking and inhaling irritants/pollutants as well as recurring infections.

With chronic bronchitis, the bronchial tree epithelium’s cilia becomes damaged - it no longer is able to properly move mucus out - and this increases mucus producing goblet cells, enlarged mucus glands, and squamous cell metaplasia. This can lead to excessive mucus production and retention which leads to trapped bacteria, bronchial wall thickening, and fibrosis. This leads to narrowing of the bronchioles that leads to obstruction.

The signs and symptoms of chronic bronchitis include chronic cough, purulent sputum, dyspnea, wheezing, hypoxemia, hypercapnia, cyanosis. (Blue Bloater)

Chronic bronchitis and emphysema is often seen concurrently.

COPD is diagnosed via spirometry, chest x rays, and lab tests to diagnose polycythemia or the increase in red blood cells due to hypoxia.

Treatment includes smoking cessation to decrease progression, bronchodilators, mucolytic agents, anti-inflammatory, and oxygen therapy.

221
Q

What is the leading risk factor for COPD?

A

Smoking

222
Q

What are the characteristics of asthma?

A

Asthma is a chronic obstructive respiratory disease caused by bronchospasm, mucous, and edema narrowing airways leading to increased resistance.

Its etiology can be a genetic influence (particularly in children) with an IgE hypersensitivity reaction or an environmental influence like respiratory infection, smoking, irritants/pollutants, stress, fatigue, endocrine changes, or weather/temperature changes.

The mechanism includes either a genetic or environmental factor triggering a hypersensitivity reaction. The early response includes mast cells releasing histamine and inflammatory mediators which create an inflammatory environment in the respiratory system which leads to edema, mucus plugs, and bronchospasms. This leads to airway obstructions and then the path of asthma.

The delayed response (6-24 hours) involves a cytokine release and eosinophil migration along with other cells of chronic inflammation. Their presence in the airways is going to cause destruction of the epithelium. The smooth muscle can have hyperplasia and it can cause narrowing of the airways and bronchial constriction.

The signs and symptoms of asthma include wheezing, breathlessness, chest tightness, sputum production, anxiety, tachypnea (increase in respiratory rate), acidosis, and hypoxia.

Diagnosis can be difficult because the patient might not be symptomatic when they’re with the clinician, but we can look for respiratory distress, pulsus paradoxus, wheezing enlongated expiratory phase, family history of hypersensitivities, and you can do labs to check for increased eosinophils, blood gas measurements of hypoxemia or hypercapnia, and peak expiratory flow rate.

Treatment includes elimination of the triggering allergen, bronchodilators, and anti-inflammatories.

223
Q

What is atopy?

A

Atopy is the genetic tendency for developing IgE-mediated hypersensitivity reactions in response to environmental allergens. It is one of the strongest predisposing factors for developing asthma.

224
Q

What is pneumothorax?

A

Pneumothorax is the presence of air in the pleural space that causes a partial or complete collapse of the affected lung.

Spontaneous pneumothorax is from the rupture of an alveolus or an air filled bleb on the surface of the lung. The air moves from the outside into the pleural cavity after the rupture and the lung collapses because of its own recoil in the absence of the tension caused by the pressure gradient in the pleural space cavity. There is also traumatic pneumothorax caused by a penetrating injury. Additionally, tension pneumothorax is when air enters the pleural space but cannot exit such as a penetrating chest wound.

The signs and symptoms include chest pain, increased respiratory rate, and dyspnea. With tension pneumothorax, the structures will be shifted to the unaffected side - the trachea and mediastinum will be deviated outside of the midline.

Pneumothorax is diagnosed by chest xray or chest CT scan. Pulse oximetry and arterial blood gas can also determine blood oxygenation.

Treatment of pneumothorax includes repairing the wound, removing air via needle aspiration or drainage system, reduce obstructions or compressions, re-inflate the collapsed area, and oxygen administration if hypoxemia needs to be treated.

225
Q

What is atelectasis?

A

Atelectasis is incomplete expansion of the lung or portion of the lung due to airway obstruction, compression, pleural effusion, or decreased surfactant. Airway obstructions are common from mucus plugs or foreign objects that make it down into the bronchial tree. The air in the alveoli gets reabsorbed so there’s no further incoming air downstream of the obstruction, so the lung will collapse. External compression can be from a tumor or fluid mass in the pleural cavity.

Signs and symptoms include tachypnea, tachycardia, shortness of breath, cyanosis, hypoxemia, and a decrease in chest expansion.

Atelectasis is diagnosed by signs and symptoms and chest xray or chest CT scan.

Treatment is dependent on the cause and extent of lung involvement. If possible, treatment will reduce the airway obstruction or lung compression and re-inflate the collapsed area. Oxygen administration, ambulation, deep breathing, and body positions that favor increased lung expansion are helpful treatments.

226
Q

What type of substance causes a pulmonary embolism?

A

The embolism may be a thrombus, air accidentally injected into an intravenous infusion, fat from the bone marrow after a fracture or trauma, or amniotic fluid that enters the maternal circulation after rupture of membranes.

227
Q

What is tidal volume? (VT)

A

Tidal volume is the normal volume of air inhaled or exhaled with each breath, ~500mL

228
Q

What is the inspiratory reserve volume (IRV)

A

The inspiratory reserve volume is the amount of air that can be forcibly inspired after taking in a normal breath (VT), ~3100mL

229
Q

What is the expiratory reserve volume (ERV)

A

Expiratory reserve volume is the amount of air that can be forcibly exhaled after letting out a normal breath (VT), ~1200mL

230
Q

What is the residual volume (RV)

A

The residual volume is the air remaining in the lung after forced expiration, ~1200mL.

231
Q

What is vital capacity (VC) and how is it calculated

A

Vital capacity is the amount of air that can be exhaled following a maximum (forcible) inhalation, ~4800 mL
VC = VT + IRV + ERV

232
Q

What is inspiratory capacity (IC) and how is it calculated

A

Inspiratory capacity is the max amount of air that can be inhaled following a normal expiration, ~3600 mL
IC = VT + IRV

233
Q

What is functional residual capacity (FRC) and how is it calculated

A

Functional residual capacity is the amount of air that remains in the lungs after a normal expiration, ~2400 mL
FRC = RV +ERV

234
Q

What is the total lung capacity (TLC) and how is it calculated

A

Total lung capacity is the sum of all the lung volumes, ~6000 mL
TLC = IRV + VT + ERV + RV

235
Q

What are pulmonary function tests (PFTs)

A

Pulmonary function tests look at pulmonary flow rates in relation to time.

Maximum voluntary ventilation (MVV): the volume of air a person can move into and out of the lungs during maximum effort lasting 12-15 seconds

Forced vital capacity (FVC): the volume of air that can be quickly and forcefully exhaled following a full inspiration (total lung capacity) - it will be low in obstructive disease.

Forced expiratory volume (FEV): expiratory volume in a given time. FEV1 is the FEV exhaled in the first second of FVC. Also used for diagnosing obstructive lung disorders.

Forced inspiratory vital flow (FIF): measures the respiratory response during rapid maximal inspiration

236
Q

Describe pulmonary embolism including the risk factors, disease pathology, clinical presentation, diagnosis, and treatment

A

Pulmonary embolism is when the pulmonary arterial bed is obstructed by a thrombus. It is most commonly caused by deep vein thrombosis, particularly in the lower extremities or pelvis. DVT is caused by immobility, traveling or sitting for many hours. It causes blood to pool in in the veins in the lower extremities so clotting factors don’t get diluted and the risk of a clot increases.

A thrombus in the vein breaks away and becomes an embolism. It’s going to continue to move into larger veins towards the heart and won’t really get stuck until the smaller arterial vessels. The embolism will either dissolve, fragment, or continue to grow. If it occludes an artery, lit leads to decreased surfactant in the alveoli and collapse of the lung. It it causes a vascular obstruction that’s large enough it can also cause an infarction and death of lung tissue.

Signs and symptoms: dyspnea, chest pain, tachycardia, feeling anxiety, cough, pleural effusion, leg edema

Diagnosed by xray and CT cans. In high risk patients, pulmonary angiography can be used - contrast dye is used along with xray to see blood flow through the lungs.

Treatment: anticoagulant and fibrinolytic drugs or surgery. Can use an umbrella filter to filter out emboli from systemic circulation and it blocks the emboli from making it to the heart and into the lungs.

237
Q

Describe ARDS (acute respiratory distress syndrome) including the risk factors, disease pathology, clinical presentation, diagnosis, and treatment

A

ARDS is an acute lung injury. It’s characterized by severe inflammation and pulmonary edema without fluid overload or abnormal cardiac function. It decreases compliance in the lungs and paraventilation and decreased oxygen levels.

It only takes 24-48 hours from an injury for ARDS to take place and the mechanism is similar to the inflammatory response. An injury causes a release of inflammatory mediators which cause the capillaries to become permeable and that will allow fluids to infiltrate into the interstitial space and then into the alveoli. This is going to lead to a very protein rich edematous fluid that’s going to lead to an osmotic fluid pooling more fluid into the space. Activated neutrophils will release products that damage the alveolar cell. There’s going to be damage to the surfactant releasing cells and that decreases surfactant, which can cause the alveoli to collapse. The fluid in collapse is going to inhibit gas exchange and decrease lung compliance, which will eventually affect lung ventilation. At first oxygen is not going to be able to cross the respiratory membrane efficiently, but CO2 will still be able to be expired because it is more soluble.

Patients first present with alkalosis, and with time the CO2 will eventually accumulate in the blood and that leads to acidosis.

Most common causes are shock, sepsis, and trauma. Also anaphylaxis, aspiration of gastric acids, pneumonia, drug overdose, near drowning, and leukemia.

Signs and symptoms include dyspnea, hypoxemia. If it progresses - hypotension, decreased urine output, and acidosis. Eventually ventricular fibrillation. Also retractions, crackles, restlessness, and tachypnea.

Diagnosis can occur with arterial blood analysis and with looking at blood pH.

Treatment: address the underlying cause, hypoxemia, and respiratory acidosis, intubation and mechanical ventilation. Some patients recover but some have permanent lung damage.

238
Q

Describe respiratory acidosis including the risk factors, disease pathology, clinical presentation, diagnosis, and treatment

A

The blood has a narrow pH range of 7.35-7.45. Acidosis is dropping below that range. If CO2 joins with water, it will create carbonic acid, which can disassociate into water+CO2 or bicarbonate + Hydrogen ions (H+). The H+ influences pH. So the body has a dynamic organ system that can eliminate or retain carbon (lungs) or eliminate or retain bicarbonate (kidneys).

Hypoventilation (hypercapnia) leads to CO2 accumulation and an increase in hydrogen ions, which leads to respiratory acidosis. Medullary chemoreceptors will pick up in the decrease in pH and increase the rate and depth of breathing to remove CO2 and that will cause a shift in our equation so the hydrogen ion concentration is reduced to restore pH.

Causes: If CNS disease or drug induced depression of the CNS - CNS is necessary for moving those respiratory muscles. If someone has impaired ventilatory movement - neuromuscular disease, chest injury, extreme obesity. Obstructive diseases like COPD, bronchitis, emphysema. Or cystic fibrosis.

Symptoms - can be difficult to diagnose based on symptoms because there can be underlying pathologies causing the acidosis with similar symptoms and signs. So in addition to the underlying disorder symptoms, dyspnea, somnolence, anxiety, confusion/delirium.

Diagnosis with a pH below 7.35 and a PCO2 above 45 mm Hg

Treatment: bronchodilators, oxygen, removal of obstruction, if too high potassium levels hyperkalemia treatment.

239
Q

Lung volumes can be measured using a ______.

A

Spirometer

240
Q

A 30-year-old man is brought to the ER with a knife wound to the chest. On visual inspection, asymmetry of chest movement during inspiration, displacement of the trachea, and absence of breath sounds on the side of the wound are noted. His neck veins are distended, and his pulse is rapid and thready.

What is your initial diagnosis?

What type of emergent treatment is necessary?

A

1) tension pneumothorax

2) chest tube

241
Q

An 18-year-old woman is admitted to the ER with a suspected drug overdose. Her respiratory rate is slow (4-6 breaths/minute) and shallow. Arterial blood gases reveal a PCO2 of 80 mm Hg and a PO2 of 60 mm Hg.

What is the cause of this women’s high PCO2 and low PO2?

Hypoventilation almost always causes an increase in PCO2. Explain.

A

Respiratory acidosis; with a decreased respiratory rate, you are not exhaling enough carbon dioxide and it accumulates.

242
Q

Explain why the oxygen flow rate for people with COPD is normally titrated to maintain the arterial PO2 between 60 and 65 mm Hg.

A

Their medullary respiratory center has adapted to the elevated CO2 levels and no longer responds to increases in PCO2. Therefore, a decrease in PO2 becomes the stimulus for respiration. If oxygen is given at too high of a rate, it suppresses the stimulus and the respiratory drive.

243
Q

Hemoglobin’s affinity for oxygen is decreased by ____. Name one factor:

A

Any of the following - decreased pH (acidosis), increased CO2 concentration, fever

244
Q

_____ are alveoli that are ventilated but not perfused.

A

Alveolar dead space

245
Q

List the 2 major divisions of the nervous system and the structures found in each.

A

The 2 divisions of the nervous system are the central nervous system (CNS) containing the brain and spinal cord and the peripheral nervous system (PNS) containing spinal nerves, cranial nerves, and ganglia that are outside of the PNS.

246
Q

Describe the meninges and their function.

A

The meninges are connective tissue sheaths that surround the brain and spinal cord. The pia mater is the innermost layer of the meninges. It is thin and delicate and contains surface level blood vessels that perfuse the brain and spinal cord. The second layer of the meninges is the arachnoid layer. It encases the entire CNS and is delicate, nonvascular, and waterproof. Cerebrospinal fluid (CSF) circulates in an area just below the arachnoid layer called the subarachnoid space. The third layer of the meninges is known as the dura mater. The dura mater is a strong, continuous connective tissue sheath that provides the brain and spinal cord with most of its protection.

247
Q

Describe how CSF is produced and its function.

A

CSF serves as a cushion fo rthe brain and spinal cord, protecting them from outside physical force.

It maintains a constant ionic environment that allows for the diffusion of essential nutrients, electrolytes, and metabolic waste products into the extracellular fluid surrounding the CNS neurons.

A thin layer of neuroglial cells (ependyma) line the ventricles of the brain and the central canal of the spinal cord. Specialized ependyma cells called the choroid plexus project into the ventricles and produce CSF. It flows from the lateral ventricle through the intraventricular foramen into the third ventricle, the cerebral aqueduct and into the fourth ventricle where it will exit the ventricular system and small openings called foramina allow the CSF to pass into the subarachnoid space where it bathes the brain and spinal cord.

248
Q

List all the cranial nerves and their functions

A

Cranial Nerve
Function
CNI – Olfactory Nerve
Olfaction (sense of smell)
CNII – Optic Nerve
Vision (sense of sight)
CNIII – Oculomotor Nerve
Pupillary constriction, accommodation, eye movement and eye lid movement
CNIV – Trochlear Nerve
Movement of eye down and inward
CNV – Trigeminal Nerve
Mastication (chewing) and sensation to the face, nose, and mouth
CNVI – Abducens Nerve
Movement of the eye laterally
CNVII – Facial Nerve
Facial expression as well as sensation of taste to the anterior tongue
CNVIII – Vestibulocochlear Nerve
Auditory sensation (sense of hearing) as well as balance and equilibrium
CNIX - Glossopharyngeal Nerve
Motor innervation to muscles involved in swallowing as well as the sensation of taste to the posterior tongue
CNX – Vagus Nerve
Digestion, regulation of heart rate, and sensation to the digestive tract
CNXI – Accessory Nerve
Motor innervation to cervical rotator muscles and the trapezius muscle
CNXII – Hypoglossal Nerve
Voluntary tongue movements

249
Q

What is the difference between afferent and efferent neurons?

A

Afferent neurons carry sensory information from the periphery to the spinal cord and brain while efferent neurons carry motor output from the brain and spinal cord to the periphery.

250
Q

List the neuroglial cells of the CNS and PNS.

A

Neuroglial cells are supportive cells that nurture and protect neurons. They can proliferate, but they cannot conduct nerve impulses or release neurotransmitters.

Ependymal cells line the ventricles or cavities in the brain and help to produce cerebrospinal fluid.

Microglial cells act as phagocytes within the CNS to clear debris, waste, or pathogens.

Oligodendrocytes produce myelin sheaths, wrappings around the neuron axons in the CNS. Myelin has a high lipid content (“white matter”), protects nueronal axons and increases overall speed of nerve impulse conduction.

Schwann cells serve the same role but for the axons in the PNS.

Astrocytes make little feet and wrap around capillaries to maintain the blood brain barrier in the brain, which helps to isolate the CNS from chemicals, toxins, pathogens, or hormones in the blood that might be detrimental to CNS tissue. They also provide structural support, help to regulate gases, and nutrient concentrations, and help to recycle or absorb neurotransmitters.

Also in the PNS are satellite cells which secrete a basement membrane protecting the cell bodies from the diffusion of larger molecules. They act to separate cell bodies and axons on the peripheral nerves from the connective tissue framework of the ganglion.

251
Q

Describe each phase of an action potential.

A

Phase 1: Resting Potential: During the resting phase, both sodium and potassium gates are closed.

Phase 2: Depolarization: The sodium gates open, and sodium rushes into the axon during the depolarization phase of the action potential. Voltage travels to zero and then up to +40 mV.

Phase 3: Repolarization: The sodium gates close, and potassium gates open allowing potassium to rush out of the axon. This returns a negative voltage to the inside of the axon.

Phase 4: After-polarization, also called hyperpolarization. Potassium gates are slow to close, and there is an undershoot of the potential. The voltage drops below -70mV and then returns to -70mV as the resting state is re-established.

252
Q

Describe 3 ways in which neurotransmitters are removed from the synaptic cleft.

A

A neurotransmitter can be broken down by enzymatic activity into inactive substances.

A neurotransmitter can be drawn back into the presynaptic neuron by a process known as reuptake.

A neurotransmitter can diffuse into the intercellular fluid until its concentration is too low to elicit a postsynaptic response.

253
Q

What type of EEG wave is associated with REM sleep?

A

The beta wave, greater than 13-30 Hz with low amplitude. It’s the same wave as when a person is awake with their eyes open.

254
Q

Alpha waves are characteristic of which stage of sleep?

A

Stage 1: Alpha waves are when a person has their eyes closed but are still awake.

255
Q

Compare and contrast NREM and REM sleep regarding muscle tone, vital signs, and brain activity.

A

During NREM sleep, there is decreased vital signs and brain activity and moderate motor activity.

During REM sleep, there is diminished motor activity and muscle tone, and increased vital signs and brain activity.

256
Q

A patient with diagnosis of Alzheimer’s disease is able to function independently at home, but their family reports that they have trouble planning and executing everyday tasks that used to come easy to them e.g. paying their bills. What stage of Alzheimer’s disease are they in?

A

Early or Mild Alzheimer’s

257
Q

A patient with a diagnosis of Alzheimer’s disease has no awareness of their surroundings and is completely dependent for all ADLs. What stage of Alzheimer’s disease are they in?

A

Late or Severe Alzheimer’s

258
Q

List 3 non-medicinal interventions that can be used to address sleep changes in someone with Alzheimer’s disease.

A

Keep a regular routine during the day
Encourage daily exercise that is earlier in the day and not close to bedtime
Avoid caffeine intake altogether if possible but certainly in the afternoon and evening
Discourage watching television right before bed
Make sure the room environment is familiar and comfortable for the individual
Provide the person with any security object that is a source of comfort to them

259
Q

What is the main difference between focal and generalized seizures?

A

Focal seizures affect one specific part of the brain in 1 hemisphere. Generalized seizures affect both hemispheres of the brain.

260
Q

Describe the phenomenon of aura.

A

Aura is a sensation or movement that feels different or “off,” and is often described as a warning sign that a seizure is about to occur.

261
Q

Give 2 reasons why it is important for someone with a seizure disorder to regularly see their care team (Neurologist and PCP.)

A

To make sure their current medication is managing their seizure disorder and to make sure there aren’t any new conditions that would require pharmacologic intervention that could interfere with their seizure medication.

262
Q

Parkinson’s disease is characterized by a loss of which neurotransmitter?

A

Dopamine, which is essential for planning and controlling body movement.

263
Q

What are the 3 hallmark signs of Parkinson’s disease?

A

Tremor, rigidity, and bradykinesia or slowness in planning, initiating, and execution of body movements.

264
Q

Why is Levadopa given to treat Parkinson’s disease as opposed to dopamine?

A

If dopamine is given in isolation it will not cross the blood brain barrier. Levadopa is the precursor to dopamine and it does cross the blood brain barrier.

265
Q

What are the 2 types of strokes? Describe the difference between them.

A

Ischemic strokes are the result of an infarct caused by cerebral artery occlusion from a thrombus or embolism. Hemorrhagic strokes are the result of a ruptured blood vessel causing a hematoma, edema, compression of the brain, or spasm of other blood vessels.

266
Q

What is a TIA and why are they important?

A

Transient ischemic attacks (TIAs) are often referred to as “mini strokes”. There is a temporary disruption of blood flow as with an ischemic stroke usually due to atherosclerotic blockage in the main feeding arteries to the brain. A person will experience symptoms; however, the symptoms will resolve within 24 hours without permanent damage to the brain. These should not be ignored as they serve as a warning sign. Most people who experience a TIA will eventually experience a stroke.

267
Q

What type of stroke are tPA drugs used to treat? How soon after the onset of a stroke must they be administered?

A

tPA drugs are used to treat ischemic strokes and must be administered within 3-4.5 hours after the onset of stroke symptoms.

268
Q

What are the goals of acute treatment of a hemorrhagic stroke?

A

Control bleeding and decrease pressure on the brain tissue

269
Q

List 2 factors that could contribute to depression.

A

Inherited/familiar link
Chemical imbalance
Hormonal imbalance/change
Stress/Trauma leading to an altered HPA axis

270
Q

A “nervous tick” is associated with what specific type of depression?

A

Depression with catatonic features

271
Q

Hypersomnia is associated with what specific type of depression?

A

Atypical depression - symptoms get worse as the day goes on.

272
Q

List the 4 diagnostic characteristics of insomnia.

A

One is considered to have insomnia if they experience 3 of the following:

Difficulty initiating sleep
Difficulty maintaining sleep
Waking up too early
Chronic nonrestorative or poor sleep

273
Q

What is sleep hygiene?

A

Sleep hygiene involves the establishment of consistent sleep patterns (going to bed at the same time each night and only sleeping as long as one needs to feel refreshed during the day), creating a comfortable sleeping environment (optimal room temperature), avoidance of screens and excessive light right before bed, and finally, avoidance of stimulants (caffeine) several hours prior to normal bedtime.

274
Q

What stage of sleep is impaired with a diagnosis of narcolepsy?

A

Narcolepsy seems to be an abnormal regulation of REM. People fall into REM very quickly and at inappropriate times during the day.

275
Q

What is the major difference between central sleep apnea and obstructive sleep apnea?

A

The brain does not properly signal the respiratory muscles with central sleep apnea. With OSA, the airway collapses, but the respiratory muscles maintain their function.

276
Q

List 3 risk factors for OSA.

A
Male gender
advanced age
family history
alcohol and drug abuse
obesity
large neck girth (>40 cm.)
277
Q

Describe a treatment option for moderate OSA.

A

A custom dental appliance that positions the tongue forward and protrudes the mandible can be used to maintain an open airway.
nasal/naso-oral continuous positive airway pressure (NCPAP) can be utilized during sleep. A mask is placed over the nose/nose and mouth which is connected to a machine that produces positive pressure which prevents the airway from collapse.

278
Q

A patient has experienced a seizure affecting the left temporal lobe. The family reports that the patient exhibited repetitive lip smacking and hand rubbing followed by a period of great fear and insecurity. They have experienced which type of seizure?

A

Focal seizure with impairment of consciousness or awareness

279
Q

You are seeing a patient with Parkinson’s disease in your office. You notice they have difficulty turning themselves around to sit in the chair, freezing in the process. This is an example of what physical manifestation of the disease?

A

Bradykinesia

280
Q

Sleep spindles are characteristic of which stage of sleep?

A

Stage 2

281
Q

A person reports feelings of worthlessness and guilt as well as difficulty sleeping. As they are speaking, you notice a repetitive, accentuated blinking of their eyes. What type of depression are they experiencing?

A

Depression with catatonic features

282
Q

A person reports feelings of worthlessness and guilt as well as difficulty sleeping. As they are speaking, you notice a repetitive, accentuated blinking of their eyes. What type of depression are they experiencing?

A

Depression with catatonic features

283
Q

A “drop attack” is another term for which type of seizure?

A

Atonic seizure

284
Q

_________ control the bodily response to sensory input internally and externally.

A

Interneurons

285
Q

_______ decrease the degradation of serotonin and norepinephrine.

A

Monoamine oxidase inhibitors (MAOs)

286
Q

The aim of drug therapy used for Parkinson’s disease is to increase _______ neuron activity or decrease _______ neuron activity.

A

dopaminergic, cholinergic

287
Q

The ______ houses the cerebral peduncles.

A

midbrain

288
Q

Explain the mechanism of electric synapses and how they contribute to the rapid propagation of an action potential.

A

Electrical synapses allow the passage of current-carrying ions through small openings called gap junctions. These gap junctions penetrate the cell junction of neighboring cells allowing current to flow in either direction.

289
Q

You are babysitting a 7-year-old child with a known history of seizure activity. You notice they appear flushed and seem unresponsive. You ask if they are feeling alright and they stare at you blankly. After a few seconds they act as if nothing happened and return to play. Is this a generalized or focal seizure? Based upon this description, what specific type of seizure did this individual experience?

A

Generalized, Absence seizure

290
Q

Describe the importance of brain activity that occurs during REM sleep.

A

During this phase of sleep, incoming sensory input is blocked; the brain cannot process it. However, internalized sensory tracts are stimulated allowing previously formed memories to replay in one’s mind. Studies have shown that adequate time spent in REM sleep is necessary for normal physiologic and psychologic functioning during periods of wakefulness.

291
Q

A patient reports to the emergency department where you are working at 10 pm. They report that earlier in the day beginning at approximately 9 am they began to experience tingling in their right arm. They decided to ignore it thinking it was probably a result of all the gardening they did the day before. As the day went on, the tingling sensation got worse, and they started to have difficulty speaking. The patient’s spouse reported that they were slurring their words. Past medical history includes a diagnosis of diabetes and hypercholesteremia. By the time they are evaluated, their symptoms start to subside. The tingling disappears and their speech returns to normal. What do you suspect this patient has experienced? What evidence is there to support that diagnosis?

A

I suspect the patient experienced a transient ischmic attack (TIA), which can be viewed as a “mini stroke.” These are generally the result of a partial occlusion due to atherosclerosis. With a TIA, patients may exhibit many of the telltale signs and symptoms of a stroke, but then the symptoms disappear within 24 hours. Because the patient experienced tingling, difficulty speaking, the past diagnosis of diabetes and hypercholesteremia, but then the symptoms dissapated, this sounds like a TIA. The patient should get further diagnostics done to address the blockage before it becomes a full occlusion leading to a CVA. Generally people who experience a TIA eventually experience a full CVA.

292
Q

A 75-year-old male is brought to the emergency department by their spouse at 8 pm. The spouse reports that they were cleaning up after dinner at approximately 6:30 pm, and she noticed that her husband’s speech was very unclear. When she asked him what was going on, he said that he didn’t know and that the entire right side of his face felt numb. She reports that he is very active, but he has a past medical history of atrial fibrillation. Based upon these symptoms and past medical history, what do you suspect this patient is experiencing? How would you confirm this diagnosis? What treatment should be administered? Explain why you chose that treatment.

A

This patient is more than likely experiencing an ischemic stroke. A CT scan and MRI would be needed to determine if a clot was blocking blood flow to the brain tissue and to rule out a hemorrhagic stroke. If a clot is found, this patient should receive tPA drugs as they are within the 3-4.5 hour treatment window.

293
Q

What is the diagnostic criteria for narcolepsy? Describe 2 topics of patient education you would give to your patient to help them combat symptoms of narcolepsy.

A

Narcolepsy symptoms include excesive daytime sleepiness, cataplexy, hypnagogic hallucinations, and sometimes sleep paralysis. A patient can be diagnosed after conducting a thorough patient history conducting day and night sleep studies throughout which the patient is given many opportunities for naps. If the patient falls into deep sleep in 8 minutes or less and if they go into REM sleep at least twice during nap opportunities, they can be diagnosed with narcolepsy.

I would first discuss changes that the patient could easily make immediately that do not include pharmaceuticals. The first is making sure they’re practicing good sleep hygiene like not looking at screens before bedtime, not exercising or drinking caffeine late in the day, establishing a consistent bedtime and wake time. Additionally, I would counsel them to start taking naps in the daytime.

Secondly, I would mention that stimulants like Ritalin or Adderall have proved useful to some patients to combat the daytime sleepiness, and they could be considered if needed.

294
Q

What role(s) does the liver serve?

A

It produces bile; metabolizes hormones and drugs; synthesizes proteins, glucose, and clotting factors; stores vitamins and minerals and converts them to be useable by the body; changes ammonia to urea; and converts fatty acids to ketones. The liver can store large amounts of glucose as glycogen; synthesize glucose from amino acids, glycerol, and lactic acid during times of fasting or increased demand; convert excess carbohydrates to triglycerides for storage in adipose tissue; major site for protein synthesis and degradation.

295
Q

What is the structure and function of the 4 layers of the GI wall?

A

The mucosal layer - it is made up of epithelium, connective tissue, and smooth muscle cells. It produces mucus that lubricates and protects the inner surface of the alimentary canal. It secretes digestive enzymes and substances that break down food. It absorbs the breakdown products of digestion. It maintains a barrier to prevent the entry of noxious substances and pathogenic organism.

The submucosal layer - contains dense connective tissue and some adipose tissue. Has blood vessels, nerves, and structures that secrete digestive enzymes.

The muscularis externa - consists of an inner layer of ciruclarly arranged muscle cells and an outer layer of longitudinally arranged smooth muscle layers. THese layers alternately contract to help move the contents through the GI tract via peristalsis.

The serosal layer - the outermost layer of organs and also called the visceral peritoneum. The peritoneum is the largest serous membrane in the body. It is comprised of two continuous layers, the visceral and parietal peritoneum. The parietal peritoneum lines the wall of the abdominopelvic cavity. Between the two layers is the peritoneal cavity, a potential space containing fluid secreted by the serous membranes. This serous fluid keeps a moist surface to prevent friction between the moving abdominal organs.

296
Q

What is the mesentery?

A

The mesentery is the double layer of peritoneum that encloses a portion or all of one of the abdominal viscera and attaches it to the abdominal wall. Included in the mesentery are blood vessels, nerves, and lymphatic vessels that supply the intestinal wall. It also holds the organs in place and stores fat.

297
Q

What is the omentum?

A

The omentum is a double-layered fold of peritoneum that hangs down from the stomach to adjacent organs in the abdominal cavity. The greater omentum connects the stomach and covers the transverse colon and the folds of the intestine. The lesser omentum extends between the liver and lesser curvature of the stomach. The greater omentum contains fat and has a lot of mobility to follow the movements of the intestines. It can form adhesions (fibrous scar tissue) adjacent to inflamed organs. It also cushions the abdominal organs against injury and provides insulation against the loss of body heat.

298
Q

Explain the difference between rhythmic and tonic movements:

A

Rhythmic movements are intermittent conttractions that help to mix and move food along. They are present in the esophagus, antrum of the stomach, and small intestine.

Tonic movements have a constant level of contraction or tone without regular periods of relaxation. They are present in the lower esophagus, the upper region of the stomach, the ileocecal valve, and the internal anal sphincter.

299
Q

What is the enteric nervous system?

A

The enteric nervous system is located in the wall of the GI tract and is made up of the myenteric and submucosal plexuses. These two plexuses are networks of nerve fibers and ganglion cell bodies. Interneurons connect afferent sensory fivers, efferent motor neurons, and secretory cells to form reflex circuits.

300
Q

What are the myenteric and submucosal plexuses?

A

The myenteric (Auerbach) plexus is a linear chain of interconnecting neurons located between the circular and longitudinal muscle layers. It is involved with GI motility. The submucosal (Meissner) plexus lies between the mucosal and muscle layers of the intestinal wall, and is involved with controlling secretions, absorption, and contraction of each segment of the intestinal tract.

301
Q

Explain how the stomach churns food:

A

The stomach churns and mixes food in a peristaltic fashion of 3-5 contractions per minute, each lasting 2-20 seconds. Contraction of the antrum pushes food toward the closed pyloric sphincter. Larger particles are returned to the body of the stomach for further churning. The other contents are emptied into the duodenum between contractions.

302
Q

Describe the 2 patterns of contraction in the small intestine:

A

Segmentation waves are slow contractions of the circular muscle layer. They occlude the lumen and push the contents forward and backward. Segmentation waves use 1 to 4 cm of intestine at a time. During this activity, chyme is mixed with the digestive enzymes from the pancreas and all surface area is exposed to the intestinal surface for absorption.

Peristaltic movements are rhythmic movements designed to propel the chyme along the small intestine toward the large intestine. Peristaltic movements contract, then relax, always in one direction.

303
Q

Explain the 2 patterns of motility in the large intestine:

A

The first include segmental mixing movements called haustral churning, since they occur in compartments called haustra. These movements fill and expel contents in the haustra, to ensure all surface area of the fecal mass are exposed to the intestinal surface.

The second type are the propulsive mass movements, in which a large segment of the colon (≥20 cm) contracts as a unit, moving the fecal contents forward. These mass movements last about 30 seconds, followed by a 2-3-minute period of relaxation, then another contraction.

304
Q

Explain how the GI tract is involved in both endocrine and paracrine regulation

A

Alhough the GI tract is the largest endocrine organ in the body, it is involved in both endocrine (distant) and paracrine (cell-to-cell) regulation. Endocrine regulation begins by releasing a hormone (or protein) into the blood stream. Once in the circulatory system, it travels until reaching the appropriate target cell, which then responds by releasing another hormone or chemical messenger. For example, once stomach acid (present in chime) enters the intestine, it stimulates the release of secretin. In contrast, paracrine regulation only occurs locally, for example a hormone reaches a target cell by crossing a neighboring cell membrane.

305
Q

What is CCK

A

It is a hormone in the duodenum and jejunum.

It is stimulated by products of protein digestion and long chain fatty acids.

It stimulates contraction of the gallbladder & secretion of pancreatic enzymes; slows gastric emptying; inhibits food intake.

306
Q

What is gastrin

A

It is a hormone in the antrum of the stomach and duodenum.

It is stimulated by vagal stimulation; epinephrine; neutral amino acids; calcium containing foods; alcohol. Secretion is inhibited by acid content of stomach antrum (pH<2.5).

It stimulates secretion of gastric acid and pepsinogen; increases gastric blood flow; stimulates gastric smooth muscle contraction & growth of gastric & intestinal mucosal cells

307
Q

What is ghrelin

A

It is a hormone in the fundus of the stomach. It is stimulated by nutritional fasting and decreased levels of growth hormone.

It stimulates secretion of growth hormone; acts as an appetite stimulating signal from stomach when an increase in metabolic efficiency is necessary.

308
Q

What is GLP-1

A

It is a hormone in the distal small intestine.

It is stimulated by a high carb meal.

It stimulates augmentation of insulin release; suppresses glucagon release; slows gastric emptying; decreases appetite and body weight

309
Q

What is GIP

A

It is a hormone in the small intestine, mainly jejunum

It is stimulated by a high carb meal

It augments insulin release

310
Q

What is secretin

A

A hormone in the duodenum

It is stimulated by acid pH or chyme entering the duodenum (pH<3)

It inhibits gastric acid secretion

311
Q

What are the three main cells in the stomach necessary for digestion?

A

The parietal cells (HCl and intrinsic factor), chief cells (pepsinogen), and G cells (gastrin).

312
Q

What are the different types of diarrhea?

A

Acute: less than 14 days; infectious microbes, self limiting, will clear on own.
Acute - noninflammatory (large volume): pathogens secrete toxins that promote fluid retention which increase volume of stool.
Acute - inflammatory (small volume): dysentery; c difficile or E coli; injury and inflammation to the eipthelium, bloody stools, fever.

Chronic: 4+ weeks; assoc with IBS, IBD, malabsoprtion syndromes, and endocrine disorders
Chronic osmotic - osmotic gradient disrupted and lumen contents have hyperosmolarity so it pulls water into the small intestine that cannot be absorbed by the colon; lactose intolerance
Chronic secretory - secretory processes of the small intestine increase, the intestine itself releasing electrolytes causing the hyperosmolarity. Due to bacterial overgrowth, cancers, bioacids, laxatives; fat malabsorption
Chronic inflammatory - IBD
Chronic Infectious - protozoans

313
Q

How do you diagnose and treat diarrhea

A

Diagnosis - symptoms of frequent stools and history of concurrent illness, medication use, travel, exposure to potential intestinal pathogens.

Treatment - some people need nothing; fluid and electrolyte replacement. Meds include lomotil and loperamide to slow GI motility and stimulate electrolyte absorption. Pepto bismol inhibits intestinal secretions and works to decrease frequency of unformed stools. Antibiotics if pathogens identified

314
Q

Describe the different types of constipation

A

Constipation is infrequent, incomplete, or difficult passage of stools.

Common causes - failure to respond to the urge to defecate, inadequate fiber in the diet, inadequate fluid intake, weakness of the abdominal muscles, inactivity and bed rest, and pregnancy.

Normal transit - perceived difficulty in defecation and usually responds to increased fluid and fiber intake.

Slow transit - infrequent bowel movements usually caused by changes in the motor function of the colon. E.g. Hirschsprung disease, defect causes absent ganglion cells in distal bowel.

Disorders of defecatory or rectal evacuation - lack of muscle coordination in the pelvic floor or anal sphincter.

315
Q

What is diverticulosis and diverticulitis?

A

Diverticulosis refers to herniation of mucosa and submucosa through the muscular layer of the colonic wall. When these pouches become inflamed or infected, it is caused diverticulitis, which can cause lower left abdominal pain, fever, and GI bleeding.

Associated with constipation.

316
Q

What are some of the causes, diagnoses, and treatments for constipation

A

Causes - narcotics, anticholinergics, calcium channel blockers, diuretics, calcium and iron supplements, aluminum antacids. People with longstanding constipation and straining may develop dilation of the rectum and colon which can cause stool to accumulate with little or no senstation.

Diagnosis - history of infrequent stools, straining, hard or lumpy stools. Rule out other diseases. Rectal exam for fecal impaction, anal stricture, or rectal masses, and check for occult blood in stool. Colonic transit time tests for severe cases.

Treatment - responding to the urge to defecate, adequate fluid and dietary fiber intake, moderate exercise, sparing use of laxatives and enemas.

317
Q

What is Irritable Bowel Syndrome?

A

IBS symptoms include recurrent abdominal pain or discomfort that is associated with a change in stool frequency or form. The pain or discomfort may be relieved by defecation. The pain is usually intermittent, cramping, and in the lower abdomen. There may be varying complaints of flatulence, bloating, nausea and anorexia, constipation or diarrhea, and anxiety or depression.

318
Q

What is the etiology, diagnosis criteria, and treatment of IBS

A

Etiology: altered transit due to muscle changes in GI tract; infection (giardia or gastroenteritis) might change the microflora in the gut; immunologic changes increase inflammation; neural changes cause dysregulation between the enteric and CNS in its modulation of the GI tract processes; food, hormones, stress.

Diagnosis: patient history
Rome criteria - abdominal pain and discomfort at least 1 day a week in the last 3 months; pain related to defecation; altered frequency and consistency of stools.
Continuing or recurrent symptoms of at least 12 weeks duration of abdominal discomfort or pain in the preceding 12 months, with 2/3 features - relief with defecation, onset associated with a change in bowel frequency, and onset associated with a change in form of stool.
Lactose intolerance

Treatment - reassurance, stress management, increased fiber, avoid offending foods, antispasmodic or anticholinergic drugs.

319
Q

What is inflammatory bowel disease

A

Crohn’s disease & ulcerative colitis. Inflammatory response, systemic symptoms, familial occurence.

Etiology: genetic predisposition, immune dysfunction, and environmental factors.

Both result from inflammatory cells and mediators that cause tissue damage.

320
Q

What is Crohn’s disease

A

Form of IBD - an inflammatory response that can affect any part of the GI, mostly the terminal ileum or cecum. It is a recurrent, slowly progressive disease that affects people in their 20-30s, and more women.

Granulomatous lesions surrounded by normal mucosal tissue. Skip lesions.

All layers of the bowel are affected; has a cobblestone appearance from fissures and crevices that develop around the submucosal edema. Pathology of the submucosal layer includes inflammatory and fibrotic changes; the bowel wall becomes thickened and inflexible; the adjacent mesentery becomes inflames; regional lymph nodes and channels may become enlarged.

321
Q

Describe the clinical presentation, diagnosis, and treatment of Crohn’s Disease

A

Exacerbations & remissions.
Main symptoms depend on involved locations - diarrhea, abdominal pain, weight loss, fluid and electrolyte disorders, malaise, low grade fever.
Less bloody diarrhea than ulcerative colitis due to it affecting the submucosal layer more than the mucosal layer. Severe diarrhea causes ulceration of the perianal skin. Nutritional deficiencies may occur due to damage in the absorptive surface of the intestines.
Fistulas can form, abdominal abscesses, and intestinal obstruction.

Diagnosis - visualize small intestine with a capsule endoscopy; colonoscopy for colon; stool cultures for infectious causes; CT scans for inflammatory mass or abscess.

Treatment - stopping inflammatory response, promoting healing, maintaining adequate nutrition, preventing and treating complications. Medications - suppress inflammation like corticosteroids, immunosuppresants, and immunomodulators. Surgical intervention for damaged bowel, drainage of abscesses, or repair of fistula tracts. Fatty food should be avoided.

322
Q

What is ulcerative colitis

A

Ulcerative colitis is an inflammatory disease of the colon, common in US and Western countries. Affects the rectum and colon only. The inflammation is continuous. The ulcers form in the crypts of Lieberkuhn which lead to the formation of pinpoint mucosal hemorrhages and end in crypt abscesses. The lesions can become necrotic and ulcerate. The mucosal layer can develop tongue like projections that resemble polyps called pseudopolyps. Because of inflammation, the bowel wall thickens.

323
Q

Describe the clinical presentation, diagnosis, and treatment of UC

A

Clinical presentation: relapsing bouts of diarrhea usually with blood and mucus. Mild abdominal cramping, incontinence, anorexia, weakness, and fatigue.
Severity is defined as mild, moderate, severe, or fulminant dpeending on how much of the colon is affected and the extent of inflammation. People with fulminant disease are at risk of toxic megacolon, which is dilation of the colon with signs of systemic toxicity due to the vast inflammatory response.
Other complications include perforation, homorrhage, and colonic carcinoma.

Diagnosis: sigmoidoscopy, colonoscopy, biopsy, negative stool cultures for infectious disease.

Treatment: dependent on severity, symptoms, geared to control acute manifestations and prevent recurrence. Mild to moderate can avoid irritating foods. Fiber supplements to decrease diarrhea. Meds like corticosteroids, immunosuppresants, and immunomodulators. Surgery to remove rectum and colon if needed.

324
Q

Explain the process of neurotransmitters activating in the GI tract

A

The stomach produces a proenzyme called pepsinogen that is activated by gastric juice to become pepsin. Pepsin starts to digest proteins in the stomach. The gastric juice is made of hydrochloric acid. It’s produced by parietal cells in the walls of the stomach.
Numerous neurotransmitters like histamin, gastrin, and acetylcholine bind to receptors on the parietal cells and that leads to intracellular signaling pathways that are going to activate the proton pump. The proton pump is going to be moving hydrogen ions into the lumen of the stomach, where they’re going to be able to bind to chloride ions and form the hydrochloric acid which has a very low pH. Because there are neurotransmitters involved, we can see that the nervous system is helping to guide this process in digestion.
This acid and pepsin would actualy damage the cells that are releasing it. The pepsin could digest proteins in the surface of these cells. And the acid would destroy these cells. Thus the stomach cells have a mucus lining that’s protecting them called the mucosal barrier.
So along with the cells for the neurotransmitters, these stomach cells have receptors for prostaglandin B2, which inhibits stomach acid production and encourages the secretion of mucus.
Drugs like NSAIDs can disrupt that prostaglandin signals and make patients prone to ulcers and gastric bleeding because we decrease the mucosal barrier.

325
Q

What is GERD

A

GERD is the severe and frequent backwards movement of gastric contents into the esophagus, occurring at least 2x/week.

The esophagus enters into the cardiac region of the stomach close to the heart, which is why it feels like the chest is burning. Can affect people after they have a large meal, eat spicy or acidic foods, consume too much coffee or alcohol. The lower esophageal sphincter relaxes from stomach distension or high fat meals and then allows the stomach acid to reenter the esophagus. The issue is corrected by esophageal peristalsis and salvia neutralizes the acid (bicarbonate).

Two types of GERD:
Erosive esophagitis - mucosal damage to the esophagus
Nonerosive Reflux Disease (NERD) - no mucosal damage

Complications: bleeding and ulcers, strictures (narrowing esophagus from repeated injury and scaring), Barrett Esophagus - The normal squamous mucosa that lines the esophagus is gradually replaced by abnormal columnar epithelium (as seen in the stomach or intestines). Barrett esophagus is a major risk factor for developing esophageal adenocarcinoma.

326
Q

How do you diagnose and treat GERD

A

Belching, chest pain, dyspahgia, bloating or early satiety, laryngitis, chronic cough, sour taste in mouth, dental erosion. Alarming symptoms are weight loss, persistent vomiting, dysphagia, odynophagia, or blood in stool.

Patient history
Acid suppression trial - proton pump inhibitor for 14 days
Esophagoscopy
Esophageal pH monitoring

Treatment:
Lifestyle changes: lose weight, avoid aggravating foods, eat smaller meals, don’t laydown for 3-4 hours after meals, elevate bed
Pharmaceuticals: antacids (alkaline, neutralize stomach acids), H2 receptor antagonists (histamine), proton pump inhibitors (limit production of stomach acid), gastrointestinal stimulants (increase motility of stomach to prevent damaging gastric juices from reaching the esophagus)

327
Q

What are the 2 most common causes of peptic ulcer disease?

A

The two most common causes of PUD include H. pylori infection and the use of aspirin and other NSAIDs.

328
Q

What is peptic ulcer disease (PUD)

A

Peptic ulcers are open sores in mucosal lining of stomach and duodenum. H. Pylori causes most of them via inflammatory mechanisms, cytokines contribute to mucosal damage. Duodenal (more common) and gastric ulcers.
NSAIDs or steroids inhibit the production of prostaglandins that generate the mucosal barrier in the stomach.
Zollinger-Ellison syndrome - hypersecretory state where too much stomach acid is secreted because of a gastrin secreting tumor.
Smoking, alcoholi, caffeine, emotional stress, genetic predisposition.

Once a patient has an ucler they can reoccur because of incomplete healing or scar tissue. Ulcers can go through any tissue in the GI tract wall and if it goes through smooth muscle, it does not regenerate completely which can lead to further exasperations.

329
Q

How do you diagnose and treat PUD

A

Symptoms: many are asymptomatic, pain that in upper abdomen, burning, cramp like. Gastric ulcers are worsened by eating; duodenal are worse when stomach is empty. Pain may be present for weeks and then remit.

Complications: gastroduodenal bleeding (tissue bleeding or erosion of an ulcer into artery or vein); weakness, dizziness, dehydration, cool moist skin, dark tarry stools. Perforation is less frequent but life threatening (when ulcer erodes through all layers and GI contents enter the peritoneum and cause peritonitis - abdominal pain radiating into the back, night distress, inadequate pain relief - appear on xrays).

Diagnose: endoscopy to visualize the ulcer, biopsies for H pylori, and exclude malignancies. History to evaluate for NSAID use. Lab results for anemia and check stool for occult blood. Barium radiography if needed.

Treatment: eradicate cause and heal the ulcer. PPI, antibiotics if needed. Prostaglandin analog for NSAID associated ulcers to stimulate mucus and bicarbonate secretion.

330
Q

What is the final product of heme breakdown?

A

Bilirubin

331
Q

What is jaundice

A

Jaundice is caused by high levels of bilirubin in the blood and is associated with yellowing of the skin, sclera, and mucus membranes (hyperbilirubinemia).

It is an issue of another liver disease - treat the liver disease not the jaundice.

As red blood cells breakdown, the heme in hemoglobin is metabolized into bilirubin which can be toxic, has a yellowish pigment, and it gets processed to the liver where it binds to bile to be eliminated via the stool. When red blood cells are destroyed, the final product of heme breakdown is bilirubin. This free bilirubin, which is insoluble in plasma, is transported in the blood attached to plasma albumin. As it goes through the liver, it is released from albumin and converted to conjugated bilirubin, which is soluble in bile. In the intestine, conjugated bilirubin is converted into a highly soluble urobilinogen by the intestinal flora. Approximately one fifth is absorbed into portal circulation while the remaining amount is excreted in the feces. Normal serum bilirubin is a small amount, less than 1.5 mg/dL.

Pre-hepatic (excessive red blood cell destruction) - hemolytic blood transfusion reaction, sickle cell disease, thalassemia, acquired or autoimmune hemolytic disorders, hemolytic disease of the newborn.
Intrahepatic (with the liver itself) - decreased bilirubin uptake by liver or decreased conjugation, liver damage via hepatitis, cirrhosis, cancer, drug induced cholestasis.
Post-hepatic (obstruction of bile flow) - bile duct structural disorders, cholelithiasis, congenital atresia of the extrahepatic bile ducts, tumors blocking the bile duct.

332
Q

What is neonatal jaundice

A

Hyperbilirubinemia caused by increased production of bilirubin and the neonate’s inability to excrete it. Greater risk of neurotoxicity - encephalopathy, cerebral palsy, sensorimotor deficits, kernicterus (brain damage from bilirubin accumulation in the basal ganglia and brainstem)

Etiology:
Physiologic hyperbilirubinemia - most infants; the bilirubin increases with the destruction of red blood cells and those have a shorter lifespan in neonate. Less efficient at removing the bilirubin because there is going to be more enterohepatic circulation (bile reabsorbed in liver, resecreted into the bile)
Breastfeeding jaundice - take in less milk in beginning, dehydrated, increase enterohepatic circulation
Breast milk jaundice - molecule in milk increases bilirubin reabsorption
Liver disease
Pathologic hyperbilirubinemia from hemolytic disease (red blood cells destroyed, increase heme), hypothyroidism, sepsis, or hematoma resportion.

Treatment based on bilirubin level, age, prematurity, health. Phototherapy - using white or blue light to photo-isomerize unconjugated bilirubin into a water soluble form that can be excreted by the liver and kidneys.

333
Q

How do you diagnose and treat jaundice

A

Lab liver function tests to diagnose liver disease to look for enzymes ALT and AST. ALT is liver specific, both are elevated in liver damage. Serum bilirubin, GGT, alkaline phosphatase measure hepatic excretory function.

334
Q

What is cirrhosis

A

Cirrhosis is the process of continued liver damage and injury leading to fibrosis and nodules that decrease the normal functions of the liver tissue. Many chronic liver disease lead to cirrhosis. Some patients might be asymptomatic and have typical life expectancy, some are at the end stage of liver disease.

Cirrhosis is characterized by diffuse fibrosis and nodules that cause scarring which disrupts blood and bile flow in the liver. Disruption of blood flow leads to portal hypertension; disruption of bile flow causes bile stasis and a loss of liver cells, leading to liver failure. Cirrhosis is associated with an increased incidence of hepatocellular carcinoma.

Nonalcoholic fatty liver disease is now the most common cause of chronic liver disease in the Western world and is associated with obestiy and metabolic syndrome.

Etiology: viral hepatitis (BCD), alcoholic liver disease, non alcoholic fatty liver disease (NAFLD).

Clinical manifestations: decreased liver function, fatigue, anorexia, weight loss, muscle wasting, jaundice, purpura, portal hypertension and can see veins around stomach, aceites, edema in lower limbs, testicular atrophy leading to gynecomastia, hemorrhoids. Hepatomegaly, jaundice, with abdominal pain most common signs.

Diagnosis: history and physical exam, full panel of blood tests, upper GI endoscopy to rule out other causes of disease. Signs of advanced cirrhosis detected using ultrasound, CT scan, and MRI. Liver biopsy most specific and sensitive test.

Treatments: aimed at controlling symptoms and trying to stop progression. Antivirals for hepatitis. Steroids for autoimmunity.

335
Q

What are the common causes of cirrhosis?

A

Cirrhosis can develop from viral hepatitis, toxic reactions to drugs and chemicals, biliary obstruction, non-alcoholic fatty liver disease, but is most commonly associated with alcoholism. Cirrhosis can also occur from metabolic disorders that cause deposition of minerals in the liver.

336
Q

What is nonalcoholic fatty liver disease?

A

NAFLD is now the most common cause of chronic liver disease in the Western world. It is associated with obesity and metabolic syndrome (type 2 diabetes, hyperlipidemia).

337
Q

What is cholelithiasis and what are they primarily made of?

A

Cholelithiasis are gall stones and they are primary made of cholesterol.

338
Q

What is cholecystitis

A

The gallbladder is a pear-shaped muscular sac located on the ventral surface of the liver. It functions to store and concentrate bile. Bile contains bile salts, cholesterol, bilirubin, lecithin, fatty acids, and the water and electrolytes normally found in plasma.
When food enters the intestine, the gallbladder contracts and the sphincter of the bile duct relaxes, so that bile can flow from the gallbladder to the duodenum. The GI hormone, CCK, is released during food digestion and aids in stimulating gallbladder contraction.

Inflammation of the gallbladder with more than 95% caused by gallstones (cholelithiasis). Gallstones block the cystic duct causing inflammation and fluid to gather in the lumen of gallbladder. The stones form because of biliary sludge becoming static or not moving as it should due to patient becoming pregnant, being given nutrition intravenously.

Cholesterol stones most common. Also black and brown stones. The two major factors that contribute to gallstone formation are increased cholesterol in bile and stasis of bile. Also 40s, obesity, female (esp multinup and oral contraceptives)

Acute cholecystitis is diffuse inflammation of the gallbladder. Eighty-five to ninety percent of acute cholecystitis is due to obstruction of the gallbladder outlet from gallstones. The other cases are due to sepsis, severe trauma, or infection of the gallbladder.
Chronic cholecystitis occurs from chronic irritation by stones or multiple attacks of acute cholecystitis. The presence of gallstones and chronic inflammation of the gallbladder are commonplace with this condition.
When gallstones block the common bile duct (the duct shared by the gallbladder and pancreas), acute pancreatitis can occur. Acute pancreatitis is a reversible inflammatory condition commonly brought on by gallstones or alcohol abuse. It presents with mid-epigastric or left upper quadrant pain that radiates to the back, with nausea and vomiting. When damaged, the pancreas will release amylase and lipase – these lab values will be elevated during this process.

Symptoms include vomiting, anorexia, malaise, fever, pain or tender upper right quadrant. Gallstones cause symptoms when they obstruct bile flow or cause inflammation. Small stones (< 8 mm in diameter) pass into the common duct, producing symptoms of indigestion and biliary colic. Larger stones are more likely to obstruct flow and cause jaundice. The pain of biliary colic presents as right upper quadrant or epigastric pain, sometimes radiating to the back or shoulder. Pain is acute, lasting more than 3-6 hours and is often associated with mild fever, anorexia, nausea, and vomiting. Lab values will often show an elevated white blood cell count, and mild elevations of AST, ALT, alkaline phosphatase, and bilirubin. With chronic cholecystitis, the patient may have an intolerance to fatty foods, belching, and colicky pain.

Worrying complication is perforation of the gallbladder.

Diagnosis via ultrasound and CT scan.

Treatment: pain management, antibiotics, cholecystectomy.

339
Q

The ______ controls the rate of stomach contents into the small intestine.

A

pyloric sphincter

340
Q

What nerve controls parasympathetic innervation to the stomach, small intestine, cecum, ascending colon, and transverse colon?

A

Vagus nerve; Cranial Nerve X

341
Q

What is it called when there is abnormally fast emptying of hyperosmotic gastric secretions?

A

Dumping syndrome

342
Q

What are slow contractions of the circular muscle layer that push the intestinal contents forward and backward?

A

These are the segmentation waves that produce the slow contractions.

343
Q

What is the purpose of propulsive mass movements?

A

Propulsive mass movements move feces towards the rectum and anus for expulsion. This movement triggers the sensation of a bowel movement.

344
Q

What is the most specific and sensitive test for the diagnosis of cirrhosis?

A

Biopsy of the liver.

345
Q

Cholelithiasis would be an example of what type of jaundice?

A

Post-hepatic jaundice.

346
Q

Excessive red blood cell destruction would cause what type of jaundice?

A

Pre-hepatic jaundice is caused by excessive red blood cell destruction.

347
Q

Describe the location of the hilus and its significance.

A

The hilus is on the kidney’s medial surface; it is a concave cleft, and it is the point where the ureters, blood vessels, and nerves enter the kidney.

348
Q

Describe the functions of the nephron.

A

The nephron is the basic structural and functional unit of the kidney. They function to control the concentration of water and soluble materials by filtering the blood, reabsorbing needed materials and excreting waste products as urine. The nephron eliminates wastes from the body, regulates blood volume, pH and pressure, and controls the levels of electrolytes.

349
Q

Name the structures of the nephron and describe their individual functions.

A

Each nephron consists of two parts, the glomerular capsule (renal corpuscle) and the renal tubule. These two structures are connected (through the tubule) to the associated collecting ducts. The glomerular capsule (renal corpuscle) filters the blood, while the renal tubule reabsorbs needed materials, and the collecting ducts carry the remaining material away as urine to be excreted.

350
Q

Compare and contrast the renal cortex and renal medulla. Discuss the structures found in each.

A

The renal cortex houses the glomeruli and convoluted proximal and distal tubules of the nephron as well as blood vessels. Portions of the cortex known as cortical columns project through the medulla into the renal pyramids.

The inner medulla is comprised of the Loop of Henle and cone-shaped masses known as renal pyramids. Each renal pyramid forms a lobe of the kidney.

The renal pelvis is located at the centermost region of the kidney. It constitutes a funnel-shaped tube that connects to the ureter as it leaves the hilus.

There are also several extensions of the pelvis called calyces that collect urine which drain continuously into the renal pelvis and subsequently into the ureter, which transports the urine to the bladder to be stored.

351
Q

Explain the difference between cortical nephrons and juxtamedullary nephrons.

A

Cortical nephrons (85%) originate in the cortex and have shorter loops of Henle that extend only a short distance into the medulla.

Juxtamedullary nephrons make up the rest and originate deeper in the cortex, and have loops of Henle that are thinner and extend into the medulla entirely.

352
Q

Explain the differences in the two systems providing the blood supply to the nephron. How does their structure determine their role?

A

The glomerulus is located between afferent and efferent arterioles, which are high resistance vessels resulting in an extremely high pressure system which can easily force fluid and solutes out of the blood into the glomerular capillary along its entire length. This allows blood to flow into the glomerulus to be filtered.

The peritubular capillaries are low pressure vessels better suited for reabsorption as opposed to filtration. These capillaries are surrounded by tubules in their entirety allowing rapid movement of solutes and water. Efferent arterioles located deep in the renal cortex turn into long, thin-walled vessels known as the vasa recta, and they run parallel to the loops of Henle in the medullary region and assist in the exchange of solutes and water flowing in and out of the kidney.

353
Q

Name two specialized structures of the glomerular capillaries that contribute to the filtration of blood.

A

Fenestrations and podocytes

354
Q

List the 4 segments of the nephron tubule.

A

The proximal convoluted tubules (highly coiled) which drains Bowman capsule

The loop of Henle

The distal convoluted tubule

The collecting tubule which joins with other nephron tubules to collect filtrate

355
Q

What are the 3 processes involved in urine formation? Describe where they occur.

A

Filtration, reabsorption, and secretion.

Filtration takes place in the glomerulus across the very porous membrane that lies between the capillaries and the interior of Bowman’s capsule. The filtration that takes place here is mechanical and does not require energy. The fluids and the solutes (water, glucose, amino acids, and nitrogenous wastes) are forced through the membrane by the high hydrostatic blood pressure in the capillary bed. The capillary pores prevent the passage of blood cells and most blood proteins across the membrane. The loss of water from the blood plasma is prevented by retaining the plasma proteins in the capillaries since they maintain the osmotic pressure of the glomerular blood.

From Bowman’s capsule, the glomerular filtrate is transported to the tubular portions of the nephron where needed solutes and fluid are reabsorbed into the peritubular capillaries from the tubular fluid and waste products are secreted from the peritubular capillaries into the tubular fluid.

Tubular reabsorption occurs as needed substances move through membrane barriers fo tubule segments to reach the peritubular capillaries. Reabsorption of water and ions are hormonally regulated and may occur through passive or active processes. The greatest amount of tubular reabsorption takes place in the PCT. Then it moves through the loop of Henle, the DCT.

Tubular secretion involves the elimination of undesirable substances from the filtrate such as urea. The body increases the concentration of the filtrate and rids itself of excess potassium ions and drugs - secretion of bicarbonate and H+ aids in control of blood pH.

356
Q

What is the norm value for the GFR? Discuss why maintenance of this value is important.

A

The norm value for the GFR is 120-125 ml/min or 180 L/day. Maintenance of the GFT is important for adequate reabsorption of water and other nutrients from the filtrate. If the flow is too rapid, needed substances cannot be adequately reabsorbed. If it’s too slow, nearly all of it is reabsorbed, including waste products that would typically be excreted.

357
Q

List the 3 regulatory mechanisms of the GFR.

A

(1) renal autoregulation, (2) nervous system control, and (3) hormonal control.

358
Q

Describe how RAAS helps to maintain the GFR

A

Hypotension or hypovolemia (decrease in blood volume) is going to decrease the renal profusion of blood and so when we have the decrease of blood pressure, we have different receptors by the kidney that can detect it. So first, we have the blood vessels that lead into the glomerulus - afferent blood vessels. They have stretch receptors, and they can pick up on that decrease in stretch and that signals to them there is a decrease in blood pressure.
We also have chemoreceptors in the cells close to the glomerulus. These are going to be the macula densa cells. They can pick up on when there is a decrease in NaCl delivery. They are screening for the presence of those ions. And so if there’s less coming through, they detect there’s less blood/blood pressure, these signals signal to the kidney to release RENIN - the hormone that the kidney can control and release.
Renin is a prohormone, it has to be processed. The process happens in different places, so the renin substrate is going to be cleaved into angiotensin I in the bloodstream. It travels and comes to the lungs where we have ACE (angiotensin converting enzymes) that will cleave it into its final active form, angiotensin II
Angiotensin II is going to act in a few different ways:
It can trigger thirst, vasoconstriction
Releases other hormones
Angiotensin II is going to trigger aldosterone secretion which is a hormone that is going to work in the distal convoluted tubule (DCT) and take back sodium. As we take back sodiu, water is going to follow it by those osmotic principles, so we will reabsorb more water. As we do so, we’ll have an increase in our blood volume and hopefully raise the blood pressure.
Agiontensin II also causes our antidiuretic hormone to be released which will cause water to be reabsorbed as well. And Angiotensin II via vasoconstriction is also going to work to raise the blood pressure.
As these mechanisms take place, there will be a decrease of renin released because we’ll be coming back into our homeostatic balance, but as the blood volume and pressure increase, we will be able to maintain that GFR, which is so important for the kidneys and their function.

359
Q

Describe the action of ADH

A

ADH acts on the collecting tubules to increase water absorption. ADH inhibits urine output by increase the number of water channels in the cell membrane of the collecting ducts.

Antidiuretic hormone (ADH)- made by the hypothalamus and it is released by the posterior pituitary gland. It’s triggered by an increase in the blood’s osmolality. It’s also triggered by the RAAS system (renin-angiotensin II - aldosterone - system) which is activated by hypotension. The angiotensin II that is produced, can cause a release of ADH. And so after ADH is released, it travels to the collecting ducts and it places pores in the cells surface, and that allows water to move from the collecting duct back into the bloodstream. As it does so it would be increasing the blood’s volume again to help raise our bloodpressure. At the same time it decreases the volume of the filtrate in urine, and by doing so it would be concentrating the urine and making it a smaller volume

360
Q

Describe the action of aldosterone

A

Aldosterone acts to place several types of ion channels inside the cells of the collecting ducts. One type of ion channel is a sodium-hydrogen ion channel. Aldosterone increases Na+ reabsorption through the excretion of H+ ions. Na+ ions are pumped out of the filtrate while hydrogen ions are pumped inside and then excreted. Because water follows salt, Na+ reabsorption will cause water reabsorption. Aldosterone will also increase K+ secretion through Na+/K+ pumps. Na+ is pumped out of the filtrate to be returned to the blood while potassium (K+) is excreted in urine. The main action of aldosterone is to increase the blood volume and, therefore, blood pressure when needed.

361
Q

List 3 ways that the kidney acts as an endocrine organ.

A

(1) through the renin-angiotensin-aldosterone system (RAA) (2) through the regulation of red blood cell production through the formation of erythropoietin, and (3) through calcium metabolism by the activation of vitamin D.

362
Q

Define hyponatremia using blood values.

A

Hyponatremia is a lack of sodium in the blood and is considered <135 mEq/L.

363
Q

Define hypernatremia using blood values.

A

Hypernatremia occurs when plasma Na+ levels rise above 145 mEq/L with a serum osmolality greater than 295 mOsm/kg.

364
Q

Define hypokalemia using blood values.

A

Hypokalemia is a lower level of potassium in the blood and is measured as <3.5 mEq/L.

Diet deficiency, diuretics, vomiting/diarrhea

Potassium is tied to aldosterone regulation, so if we’re in a hypokalemic state, our body is going to retain potassium. To do so our body is going to have to kick out sodium ions, when it does it will dilute our urine

Cardiac arrhythmia, hypotension

Dizziness, muscle weakness, leg cramps, confusion, decreased smooth muscle motility.

Hypotension, dilute urine, metabolic alkalosis

365
Q

Define hyperkalemia using blood values.

A

Hyperkalemia is an excess of potassium in the blood and is measured as >5 mEq/L.

Rarely occurs in otherwise healthy people - the body is very effective at preventing K+ accumulation in the ECF

Renal failure is most common cause

Excessive intake of potassium, release from intracellular compartments (burns and crush injuries most common reason), renal failure, use of potassium sparing diuretics or ACE inhibitors and ARBs

Cardiac arrest, abdominal cramping, flaccid paralysis

Weak muscles, dyspnea first symptoms.

Administration of calcium as it opposes the action of potassium in decreasing membrane excitability - short lived. Typically paired with other drugs including sodium bicarbonate or insulin which ac tto pull K+ into the ICF decreasing ECF concentration.

366
Q

Define hypocalcemia using blood values.

A

Hypocalcemia is a low level of calcium in the blood and is measured as <8.5 mg/L.

Impaired ability to draw calcium from bone stores - depends on adequate supply of parathyroid hormone (PTH)

Abnormal losses of Ca2+ from kidneys

Increased protein binding leading to greater amounts of CA2+ in its non-ionized form

Soft tissue sequestration

Heparin and glucagon treatment, thyroid disorders, severe burns, kidney failure, vitamin D deficiency, sepsis, acute pancreatitis

Ca2+ can only leave capillaries when in its ionized state. Changes in pH will alter the amount of calcium that is bound and in its ionized form. An acidic pH will decrease calcium binding to protein which will in turn increase its ionization. An alkaline pH will produce the opposite effect resulting in increased binding of calcium to protein and a decrease in ionized calcium

Cardiac arrhythmia, hypotension, neuromuscular irritability, anxiety, irritability, seizure, muscle twitching, cramps, spasm, tetany, laryngospasm, osteomalacia, bone pain, deformities, fracture

367
Q

Define hypercalcemia using blood values.

A

Hypercalcemia is an excess of calcium in the blood and is measured at >10.5 mg/L.

Bone breakdown, thyroid disease, calcium supplements like antacids

The two most common causes of hypercalcemia are increased bone resorption of Ca2+ due to neoplastic activity and hyperparathyroidism.

Cardiac arrhythmia, decreased neuromuscular irritability, confusion, fatigue, headache, irritability, nausea/vomiting, constipation, osteopenia, osteoporosis

Neural excitability is decreased. Stupor, weakness, muscle flaccidity. Subtle behavior changes to psychosis

Rehydration to increase excretion of calcium. ONce ECF volume has been restored, diuretics and NaCl can be utilized to increase urinary excretion of calcium

368
Q

Define hypomagnesemia using blood values.

A

Hypomagnesemia is a decrease of magnesium in the blood and is measured as 1.8 mg/dL.

Malnutrition, starvation, diarrhea leading to decrease of intestinal absorption, and alcoholism (diarrhea)

Typically occurs in conjunction with hypocalcemia and hypokalemia - similar signs and symptoms

Personality changes, tremors, tachycardia, hypertension, and ventricular dysrhthmias

369
Q

Define hypermagnesemia using blood values.

A

Hypermagnesemia is an excess of magnesium in the blood and is measured as >3.0 mg/dL.

Renal insufficiency
Disease
Overconsumption of food/supplements

Elevated levels will diminish neuromuscular functions
Hyporeflexia, muscular weakness, confusion. Blood pressure can decrease, respiratory paralysis, heart block, cardiac arrest

IV administration of calcium as it directly inhibits the effects of magnesium. In some cases dialysis for patients with kidney failure

370
Q

What are the 3 mechanisms of control for blood pH.

A

Blood pH is controlled by three main methods in the body: (1) the chemical buffer systems, (2) the brainstem respiratory center, and (3) the renal system.

371
Q

How do we diagnose acid-base imbalances

A

Arterial blood gas - blood is drawn from an artery so we measure highest content of oxygen and from that we assess pH, oxygen, CO2, and bicarbonate levels of the blood.

Anion gap (blood and urine) - assessment of different electrolytes in the blood. Cations (sodium and potassium) minus anions (chloride and bicarbonate) = the difference equal unmeasured anions that are generally phosphates, sulfates, proteins, and organic acids. It’s used for measuring metabolic acidosis.

372
Q

How do you determine if someone is in acidosis or alkalosis

A

To determine if someone is in acidosis or alkalosis you must first look at their pH level. You would then determine whether the primary cause of their disorder is respiratory or metabolic in nature by evaluating CO2 and HCO3- values. Finally, if there are multiple abnormal values, it is necessary to determine if one of the abnormal values is due to compensatory mechanisms.
Acidosis characterized by pH < 7.35 with an abnormally high PCO2 (> 45 mm) is respiratory acidosis. If renal compensation is occurring, HCO3- concentration will be > 26 mEq/L as the renal system attempts to raise the pH by increasing HCO3- concentration.
Acidosis characterized by pH < 7.35 with an abnormally low HCO3- concentration (< 22 mEq/L) is metabolic acidosis. If respiratory compensation is occurring, the PCO2 will be < 35 mm as the respiratory system attempts to raise the pH by decreasing PCO2.
Alkalosis characterized by pH > 7.45 with an abnormally low PCO2 (< 35 mm) is respiratory alkalosis. If renal compensation is occurring, the HCO3- concentration will be < 22 mEq/L, as the renal system attempts to lower the pH by decreasing HCO3- concentration.
Alkalosis characterized by pH > 7.45 with an abnormally high HCO3- concentration (> 26 mEq/L) is metabolic alkalosis. If respiratory compensation is occurring, the PCO2 will be > 45 mm as the respiratory system attempts to lower the pH by increasing PCO2.

373
Q

What are the common causes of acute postinfectious glomerulonephritis?

A

Acute Postinfectious Glomerulonephritis occurs as the result of another infection including streptococcus and staphylococcus bacterial infections, viral infections such as hepatitis, and even parasitic infections.

374
Q

Discuss risk factors for the formation of renal calculi.

A

The levels of stone components in the blood and urine, anatomical changes of the urinary tract structures, metabolic and endocrine function, dietary and intestinal absorption, past history of UTIs, and supersaturated urine.

375
Q

Be able to list and describe the 4 types of kidney stones.

A

Calcium stones - 75-80%; increased concentrations of calcium in the blood and urine. This can occur secondary to increased bone resorption, typically associated with immobility, bone disease, or hyperparathyroidism.

Magnesium ammonium phosphate stones - form in urine that is alkaline. The increase in pH is the result of a UTI caused by bacteria possessing an enzyme called urease which will break urea into ammonia and carbon dioxide. The ammonia will form ammonium ion which increases the pH of hte urine. The resulting stones increase in size as the levels of bacteria rise.

Uric acid stones coincide with gout. They form when the pH of urine is more acidic; they are not visible on xray films.

Cystine stones - rare, but most common in children. Result of cystinuria which occurs when there is a decrase of tubular absorption of cystine and is the result of a genetic defect in renal transport.

376
Q

Describe the differences between renal-colic pain and non-colicky pain.

A

Renal colic pain is described as “colicky” pain brought on by the stretching of the collecting ducts or the ureter. This type of pain occurs as stones that are 1-5 mm in diameter move through the ureter and block flow. It is an acute, sharp pain that comes in waves and is felt in the upper, lateral quadrant of the abdomen and/or the back of the affected side. This is termed “flank pain.” Pain can radiate to the lower abdominal region, bladder area, perineum, or the male scrotum. It can be accompanied by clammy, cool skin as well as nausea and vomiting. Non-colicky pain is caused by stones that distend the renal pelvis or renal calyces. It can be described as a deep, dull ache in the flank area. It varies in intensity from mild to severe.

377
Q

Be familiar with various diagnostic tests and treatments for renal Calculi.

A

Urinalysis, x-rays, CT scan, intravenous pyelogrpahy, and abdominal ultrasound.

Urinalysis looks at urine pH, stone forming crystals, infection, or hematuria.

378
Q

What are the 3 categories of acute renal failure?

A

Prerenal - most common; decreased renal blood flow.

Postrenal failure is when the outflow of urine from the kidney is impeded.

Intrarenal failure/acute kidney injury - caused by conditions that damage structures within the kidney, namely glomeruli, vessels, tubules, or the interstitium.

379
Q

What are the 2 types of ATN?

A

Ischemic acute tubular necrosis occurs most frequently among patients who have had major surgery or suffered trauma or burns. Damage to the tissue releases toxins that make the tubular cells more susceptible to ischemic effects.

Nephrotoxic ATN is brought on by exposure to drugs and other nephrotoxic agents. This exposure injures the tubules direction, through renal vasoconstriction, or by producing a tubular obstruction.

380
Q

What are the 3 phases of ATN?

A

The onset or initiating phase of ATN (hours/days) is the time between the precipitating event/exposure (i.e. ischemic event or toxin exposure) and tubular injury.

The maintenance phase is marked by a drastic decline in the GFR. This causes the retention of metabolites that are cleared by the kidney under normal conditions. Urine output is at its lowest at this point. Fluid retention will lead to edema, water intoxication, and pulmonary congestion. If this phase is prolonged, hypertension will develop as well asl uremia. If this goes untreated, seizures, coma, and even death can result.

The recovery phase of ATN is the period in which tubular repair takes place. Urine output will increase and there will be a fall in blood creatine levels. Diuresis (increased or excessive urine production) may occur during this phase and can cause excessive water and electrolyte loss. Eventually tubular function is restored as well as BUN and creatine levels.

381
Q

What blood values are closely monitored in renal failure?

A

BUN and creatinine levels

382
Q

Describe the stages of kidney failure

A

Stage 1 (>90 mL/min) - Stage 5 (renal failure, GFR<15 mL/min)

Decline in function of nephrons, healthy nephrons do pick up the slack but they’ll become over-burdened and sclerotic.

As kidneys fail to excrete sodium and water, hypertension and hypovolemia will occur which will lead to edema and heart failure.

Increase in potassium will result in hyperkalemia - nervous and muscular systems affected.

Increase nitrogenous wastes lead to uremia which affects every organ system

383
Q

Describe the process of Hemodialysis.

A

Hemodialysis utilizes an “artificial kidney” or hemodialysis system to filter the blood. It is composed of 3 parts: (1) a blood delivery system, (2) a dialyzer, and (3) a dialysis fluid delivery system. Blood circulates through the hollow dialyzer composed of bundles of capillary tubes. Dialysate (dialysis fluid) moves on the outside of the tubes. The walls of these capillary tubes are composed of semipermeable material which allows all molecules (except blood cells and plasma proteins) to move in both directions (from the blood into the dialysate and from the dialysate into the blood.) Waste products will diffuse into the dialysate. During dialysis treatments blood will flow from the patient’s artery through the blood chamber in the dialysis machine (where it is filtered) and back into the patient’s body through a vein. Vascular access is achieved via a shunt (implantation of tubing into an artery and a vein) or more commonly through an arteriovenous fistula (anastomosis of an artery to a vein.) Patients are put on blood thinners such as heparin to prevent blood clotting during treatment. Common symptoms that accompany dialysis treatment include nausea, vomiting, muscle cramps, headache, chest pain, and vertigo. Treatments are usually done 3 times each week and last 3-4 hours.

384
Q

Describe the process of peritoneal dialysis.

A

Peritoneal dialysis utilizes the same principles of hemodialysis; however, the serous membrane of the peritoneal cavity serves as the dialyzing membrane. A catheter is surgically placed in the peritoneal cavity and is tunneled through subcutaneous tissue to its exit on the side of the abdomen. A sterile dialyzing solution is run through the catheter over a period of 10 minutes. The fluid will remain in the peritoneal cavity for a prescribed amount of time allowing metabolic waste products to diffuse into the solution. At the end of this time, the fluid is drained from the peritoneal cavity into a sterile bag. This type of dialysis can be done in a dialysis clinic or at home by the patient or caregiver. One of the major concerns with peritoneal dialysis is the risk for infection at the catheter exit site.

385
Q

What are the determinants of transplantation success?

A

The overall health of the recipient, the degree of compatibility between the donor and the recipient, and the management of recipient immunosuppression.

386
Q

Under which circumstances will ADH levels rise?

A

Severe hemorrhage, excessive sweating, and diarrhea

387
Q

Excess accumulation of fluid within the interstitial compartment is characteristic of….

A

edema

388
Q

Reabsorption in the loop of Henle is characterized by:

A

The loop reabsorbs more Na and Cl than water

The acending limb is impermeable to water

Water is reabsorbed in the descending limb

389
Q

Acute postinfectious glomerulonephritis is characterized by:

A

Typically caused by a sterptococcal infection

The prognosis is good when underlying cause is treated

Accompanied by glomerular enlargement and hypercellularity

Oliguria is often the first symptom

390
Q

What is cholecalciferol

A

It is the inactive from of vitamin D taken in through the skin via UV rays

391
Q

Na+ reabsorption, through the excretion of H+ ions, is achieved through the action of what?

A

Aldosterone

392
Q

In the nephron, the mass of capillaries surrounded by an epithelial capsule that opens into a tubule is collectively referred to as the _______.

A

renal corpuscle

393
Q

The hormone that is synthesized in the kidneys and regulates the differentiation of red blood cells is _______.

A

EPO - erythropoeitin

394
Q

The enzyme ___________ causes CO2 to react reversibly with water to form carbonic acid (H2CO3) to regulate blood pH.

A

carbonic annhydrase

395
Q

A drastic decrease in urine output accompanied by a disproportionate increase of blood urea nitrogen (BUN) in comparison to serum creatinine levels is characteristic of _______ failure.

A

prerenal

396
Q

Can hyperkalemia be caused by movement of K+ from the ECF to the ICF compartment?

A

No. Hyperkalemia can be caused by the movement of K+ from the ICF to the ECF.

397
Q

Explain why renal flow is decreased with sympathetic activity.

A

Sympathetic activity diverts blood to the heart, brain, and skeletal muscles. During these times the renal autoregulatory system may be superseded by nervous system control. In this event, a narrowing of the afferent arteriole is caused by sympathetic nerve fibers followed by a release of epinephrine from the adrenal medulla which leads to a subsequent decrease in renal flow and the GFR.

398
Q

A patient has a diagnosis of acute pancreatitis and a subsequent electrolyte imbalance. Which electrolyte imbalance may they experience given this condition? Explain why pancreatitis would lead to this imbalance.

A

Hypocalcemia. Acute pancreatitis causes the release of proteolytic and lipolytic enzymes. Free fatty acids are released during lipolysis in the pancreas. Ca2+ binds to these fatty acids removing them from the blood.

399
Q

A patient has a diagnosis of glomerular disease. Given what you know about the structural framework of the glomerular capillaries, what would be detected in this patient’s urine, and explain why this would happen.

A

Blood and protein may be present in the urine. Spaces within the basement membrane of the glomerular capillaries, under normal circumstances prevent red blood cells and plasma proteins from passing through the glomerular membrane into the filtrate. The disease process would compromise this.

400
Q

Explain why a patient with chronic kidney disease may develop anemia.

A

Under normal circumstances, erythropoietin is synthesized in the kidneys and regulates the differentiation of red blood cells within the bone marrow. The formation of erythropoietin is hindered in chronic kidney disease.

401
Q

A patient presents to the emergency department with complaints of sharp pain that comes in waves in the upper lateral quadrant of the abdomen. Their skin is clammy, and they have been experiencing nausea and vomiting all day long. They have a history of UTIs. Urinalysis reveals that the pH of their urine is 7.8. A CT scan is ordered, and it reveals a stone 4 mm in diameter. What type of renal calculi do you suspect? What treatment is needed? Explain your reasoning for both answers.

A

The patient has a magnesium ammonium phosphate stone given the elevated pH of their urine. These types of stones are the result of a UTI caused by bacteria that contains urease. Urease breaks urea down to form ammonia which raises the pH of the urine. This patient has a history of UTIs so this should be taken into consideration. This patient does not require a procedure to remove the stone. Since it is less than 5 mm in diameter, the stone should pass on its own. The patient should be placed on antibiotics to treat the UTI as well as medication for pain management.

I believe they have magnesium ammonium phosphate renal calculi with renal colic pain. Ammonium phosphate renal caculi tend to develop in alkaline environments and the pH of their urine is higher than normal.

Because this is a larger stone, I would recommend uteroscopic removal - breaking the stone up into smaller pieces and manually removing them from the blockage, if possible. Additionally, since this person has a history of UTIs, I would recommend antibiotic therapy if they have an active one. Additionally, recommending taking preventative measures for the future such as improving fluid intake.

402
Q

A patient is admitted to the hospital with pneumonia. The following are the results of their blood work: pH = 6.9, PCO2 = 52 mm, and HCO3- = 30 mEq/L. Based upon these results, what type of acid-base disorder are they experiencing? Is compensation occurring? Describe a treatment intervention for this disorder. Normal values are as follows: pH = 7.35-7.45, PCO2 = 35-45 mm, HCO3- = 22-26 mEq/L.

A

The patient is in respiratory acidosis. The renal system is attempting to compensate as HCO3- concentration is elevated above normal limits. The goal of treatment for respiratory acidosis is improving ventilation. Supplemental O2 can be administered; in severe cases mechanical ventilation may be indicated.

403
Q

A 5-year-old is brought to the emergency department after spending the day at an amusement park. The child is agitated, and their skin is flushed. No tears are produced when they cry. Blood Na+ = 152 mEq/L. Based upon these symptoms and results of blood work, what electrolyte imbalance is this patient experiencing? What treatment is indicated for this imbalance?

A

This patient is experiencing hypernatremia. Treatment for hypernatremia includes treating the underlying cause and replenishing fluids orally or intravenously.

404
Q

Define endocrine, paracrine, and autocrine.

A

Endocrine: Hormones are released to circulation to act on a target organ (i.e. TSH, ADH) Paracrine: Hormones act locally on cells close to where they are released (i.e. estrogen and testosterone)
Autocrine: Hormones produce a biologic action on the cell that released them (i.e. insulin)

405
Q

What are the different structural types of hormones?

A

(1) amines and amino acids; (2) peptides and proteins; and (3) steroids, which are made from cholesterol

406
Q

What is the function of the hypothalamus?

A

The hypothalamus serves to link the nervous system to the endocrine system. It regulates homeostasis, body temperature, hunger, behavior, emotion, and pain. The hypothalamus produces releasing hormones, which stimulate the pituitary to release stimulating hormones.

407
Q

What is the role of the pituitary gland?

A

The pituitary gland is known as the master gland since it stimulates target organs to secrete their hormones.

408
Q

Explain the concept of negative feedback and give an example:

A

Negative feedback is a cycle of when the body releases a hormone and then this action sends back the message to the hormone releasing organs/glands that the action is occurring so it can turn down the hormonal message. An example of this is the action of ADH to dilute the blood. Once the blood is diute, the hypothalamus detects the dilute levels and stops releasing ADH.

409
Q

Explain the concept of positive feedback and give an example:

A

Positive feedback enhances or increases the amount of the hormone that is released. One example is oxytocin, a hormone that causes the uterus to contract. The action of the uterus contracting causes more oxytocin to be released.

410
Q

What are the components of the endocrine system?

A

The endocrine glands - groups of cells that secrete chemical substances directly into the blood or lymph. The chemical’s substances are hormones.

Hormones - chemical secretions that get produced by the endocrine glands. They can be proteins, steroids, or neurotransmitters.

Target organs - specific areas of the body that the hormones have an affect on. For a hormone to affect a target, the target cell has to have a receptor that recognizes the hormone - that helps to give specificity. Not all cells will recognize all hormones.

411
Q

What are the major endocrine glands?

A

In the brain there’s the pineal gland - secretes melatonin which helps to regulate the body’s circadian rhythm.
Also the pituitary gland - the way the central nervous system connects anatomically and functionally to the endocrine system.
And the hypothalamus - releases hormones that guide the pituitary’s release of multiple tropic hormones that guide the rest of the endocrine glands.

Thyroid gland - thyroid hormone; major regulator of all of our cell’s metabolism
Thymus - releases thymosin which assists in maturation of T cells in the thymus.
Adrenals - sit atop the kidneys; release various hormones including the catecholamines which we call adrenaline. They release corticosteroids, mineralosteroids, and gonadocorticoids.
Pancreas - releases hormones that regulate our blood glucose levels - insulin and glycogen.
Testis/Ovaries - release sex hormones that help to maintain the repro organs, give secondary sex characteristics and help to maintain pregnancy.

412
Q

What are the 4 classes of hormone based on structure

A

Amines/amino acid - two iodinated tyrosine aminos make thyroid hormone (dopamine, epinephrine, norepinephrine, thyroid hormone).

Peptide/protein - must bind to a plasma membrane receptor to have an affect on a target cell; insulin; cannot pass through the cellular membrane. Each receptor will have an affinity for a certain hormone and once the hormone binds to the receptor, it will cause a cascade of events - second messenger pathway. It causes intracellular signaling molecules to carry out some sort of cellular response (thyroid stimulating hormone, ADH, oxytocin, insulin, glucagon, corticotropin-releasing hormone, follicle simulating hormone, growth hormone-releasing hormone).

Steroids - derived from cholesterol; bind to nuclear receptors inside the target cell. The hormone can enter the plasma membrane on its own, but inside the cell it can bind to the receptor and it is then ushered into the nucleus of the cell where it interacts with the DNA and can change the transcription of the DNA to make new proteins (aldosterone, estrogen, progesterone, glucocorticoids).

Fatty acids - prostaglandins

413
Q

What are the functions of hormones

A

Open or close ion channels affecting membrane potential.
Stimulate gene expression to influence protein synthesis.
Activate or inhibit enzyme systems.
Induce secretory activity in other endocrine organs.
Stimulate mitosis for growth or healing.

414
Q

Explain the ways in which hormones can be over-secreted:

A

Increased hormone secretion can occur from any of the following reasons: (1) the target gland over-secretes due to pathology; (2) the pituitary or hypothalamus over-stimulates the target gland; (3) hormones are being produced from a different site (i.e. hormone producing tumor); (4) hyperactive genetic mutation of the target hormone receptors.

415
Q

How does decreased hormone secretion occur?

A

Decreased hormone secretion can occur from any of the following reasons: (1) a congenital or acquired disorder of the target gland; (2) the pituitary does not secrete enough stimulating hormone; (3) the hypothalamus does not secrete enough releasing hormone; (4) the hormone is defective; (5) the receptors of the target organ do not respond.

416
Q

Adrenal insufficiency can be caused by what ways?

A

Primary adrenal insufficiency - Addison’s disease - due to dysfunction in the adrenal glands.

Decreased ACTH stimulation from the pituitary, or secondary adrenal insufficicency.

Decreased CRH from the hypothalamus, or tertiary adrenal insufficiency.

417
Q

Describe the process of how hormones are released by the endocrine glands

A

The CNS takes in information internally and externally to understand the environment and integrate what the body needs and how the endocrine system can help. The higher cerebral areas send signals to the hypothalamus on how to regulate the body.
The hypothalamus will then regulate the endocrine system by communicating with the pituitary gland.
The anterior pituitary gland will send out stimulating hormones to trigger the release of additional hormones from other endocrine glands: growth hormone to simulate muscle and bone cell mitosis so those organ systems can grow; FSH and LH to the repro glands; thyroid stimulating hormone; adrenocorticotropic hormone to stimulate adrenals.

418
Q

With Addison’s disease, what hormones and lab values will be altered?

A

With Addison’s disease, both cortisol and aldosterone secretion are decreased and ACTH levels are elevated. With low aldosterone, there will be low sodium, and increased potassium. Low sodium will also cause hypotension.

419
Q

Describe Addison’s Disease, including clinical presentation, diagnosis, treatment

A

Adrenal gland

Insufficient cortisol and aldosterone secretion accompanied by elevated ACTH levels. With low aldosterone, there will be low sodium and increased potassium. Low sodium will cause hypotension.

Etiology - mostly autoimmune
Primary - destruction of the gland
Secondary - an issue at the level of the pituitary gland not releasing ACTH or the hypothalamus not releasing CRH.

Clinical manifestations - not until most of gland is destroyed.
Hyperpigmentation from high ACTH levels - similar amino acid sequence to the melanocytes.
Mineralocorticoid deficiency - affects electrolyte and water balance. Increase in sodium, chloride, and water excretion and a lowering of the blood volume, which decreases blood pressure and cardiac output. Hyperkalemia occurs without aldosterone secretion. Dehyrdation.
Glucocorticoid deficiency - hypoglycemia, weight loss, lethargy, GI symptoms like anorexia, nausea, vomiting, and diarrhea.

Treatment - hormone replacement therapy

420
Q

What can cause Cushing syndrome?

A

(1) Iatrogenic, from long-term treatment with steroids (2) Over-secretion by one or both adrenal glands (due to adrenal adenoma or carcinoma) (3) Over-stimulation of the adrenal glands by an ACTH-secreting tumor in the pituitary (this is know as Cushing’s disease.) (4) Over-stimulation of the adrenal glands by an ectopic ACTH producing tumor (most common being small cell lung cancer)

421
Q

What are the signs and symptoms of Cushing syndrome?

A

Cushing syndrome can present with a buffalo hump (fat pads) in the back, abdominal obesity, moon (round) face, muscle weakness, easy bruising, thin skin, edema, purple striae, osteoporosis and/or osteonecrosis, acne, hirsutism, virilization, immunosuppression, diabetes, and cognitive changes, including mood changes to psychosis.

422
Q

What lab values will be altered in primary and secondary hyperthyroidism?

A

With primary hyperthyroidism, TSH will be low, since the thyroid is secreting a lot of the hormone without being told by the pituitary. With secondary hyperthyroidism, the TSH will be high, since the pituitary is over-secreting it. In both cases, T4 and T3 will be elevated, since this is what differentiates hyper- from hypo- thyroidism.

423
Q

What are signs and symptoms of hypothyroidism?

A

Signs and symptoms of hypothyroidism include weight gain, fatigue, weakness, bradycardia, hypoventilation, cold intolerance, constipation, arthralgias, myalgias, and/or anemia. Goiter may be present.

424
Q

What are the roles of insulin and glucagon?

A

Insulin moves glucose into cells to reduce blood sugar levels. Glucagon causes a release of glucose into the blood when levels are low. These hormones work together to maintain normal blood sugar.

425
Q

Define glycogenesis, glycolysis, gluconeogenesis, and glycogenolysis.

A

Glycogenesis is when insulin is released from the beta cells of the pancreas to promote glucose uptake into the cells and store it as glycogen.

Glycolysis is when glucose is broken down for energy.

Gluconeogenesis is when glucagon is produced by the alpha cells, glycolysis is decreased, and glucose formation is increased.

Glycogenolysis is when glucagon is produced by the alpha cells and it decreases glycolysis and increases the breakdown of glycogen to release glucose.

Glucagon is produced by the alpha cells. It decreases glycolysis and increases gluconeogenesis, which is glucose formation and glycogenolysis, which is the breakdown of glycogen to release glucose.

426
Q

What is insulin resistance?

A

Insulin resistance is when insulin does not work effectively. Obesity leads to a decreased number of insulin receptors. This is seen in type 2 diabetes.

427
Q

What are the three types of cells in the pancreas?

A

In the pancreas’ Islet of Langerhans, there are 3 types of hormone secreting cells:
Alpha cells secrete glucagon which is considered a fasting hormone, between meals or overnight or between long periods of fasting, glucagon is secreted and its job is to increase blood glucose levels. At night when we’re sleeping our brain and tissues are still active and need an energy source. The major target of glucagon is the liver where it will trigger the breakdown of glucagon into glucose (glycogenolysis and gluconeogenesis).

Beta cells secrete insulin - the eating hormone; gets released to lower blood glucose levels after we eat a meal. It triggers the body’s cells to take up glucose, shuttles most of the glucose into fat and muscle cells, and then it inhibits the liver from the processes of glycogenolysis and gluconeogenesis. And it promotes the formation of glycogen, or the storage of this excess glucose that’s around after we eat. It also increases protein synthesis.

Delta cells secrete a hormone called somatostatin which seems to increase the amount of time nutrients are in the bloodstream so they’re more available to the body’s cells. It does this by slowing down GI tract motility and decreases insulin and glucagon secretion.

428
Q

What is diabetes mellitus

A

Diabetes mellitus are metabolic disorders where insulin secretion is not adequate to lower blood glucose resulting in hyperglycemia and inadequate energy for the body’s cells. It is due to hyposecretion or hypoactivity of insulin causing hyperglycemia. In insulin resistance, cells have a decreased response to insulin.

429
Q

What is the difference between Type I and II diabetes

A

Type 1 - autoimmune destruction of pancreatic beta cells leading to insulin deficiency (idiopathic subtype with no autoantibodies also found)
Etiology - genetic susceptibility, environmental trigger, autoimmunity destruction of beta cells so no cells to produce insulin.
The decreased insulin causes an increase in glucagon which can lead to the use of amino acids and fats as energy sources. The lipolysis creates free fatty acids that get turned into ketones by the liver which are organic acids that can yield energy. Because they are acids, leads to acidosis which if untreated can lead to CNS depression and coma. Signs: hypotension/tachycardia, fruity smell to breath, increased respiration rate, cardinal signs of DM.
Diagnosis: hyperglycemia >250 mg/dL, low serum bicarbonate (<15 mmol/L), and low pH) with ketonemia. Treat with insulin, electrolytes, fluids.

Type 2 - insulin resistance of peripheral cells and decreasing insulin secretion by beta cells over time - presents with metabolic syndrome
Etiology - obesity, physical inactivity, high calorie diet, environmental pollutants, insulin resistance

430
Q

What is metabolic syndrome?

A

Metabolic syndrome is a myriad of conditions that increases one’s risk of heart disease, stroke, and type 2 diabetes. These include elevated blood sugar, high cholesterol, hypertension, and obesity, particularly increased body fat around the waist.

431
Q

What are polydipsia, polyphagia, and polyuria?

A

Polydipsia is increased thirst.

Polyphagia is increased appetite.

Polyuria is increased urination.

432
Q

What are the primary microvascular complications of diabetes?

A

Retinopathy, which are microvascular changes in the retina that can lead to blindness; nephropathy, which is kidney disease caused by damage to the small vessels that can lead to renal failure; and neuropathy, which is nerve damage that causes weakness, numbness, or pain, typically in the hands or feet.

433
Q

What are the primary macrovascular complications of diabetes?

A

peripheral vascular disease, coronary artery disease, heart attack, and stroke.

434
Q

If a patient is taking exogenous forms of corticosteroid hormones, what would their labs show?

A

Low levels of ACTH

435
Q

What are some common signs and symptoms of Cushing syndrome

A

Poor wound healing, osteoporosis, and acne

436
Q

What are treatments of Addison’s disease

A

Hormone therapy for life, regulare mealtimes, higher doses of hormones during stress

437
Q

What hormone plays a major role in the maintenance of body metabolism and growth and development in children?

A

Thyroid hormone

438
Q

What hormones does the posterior pituitary release

A

oxytocin and ADH

439
Q

When the body is in starvation mode, what does it break down fats into?

A

Ketones

440
Q

A 45- year-old female presents with increased appetite, weight loss, and nervousness. Lab findings show a high serum T4 and low TSH.

What diagnosis would her history and lab findings indicate?
What is one treatment (not symptom treatment) you would suggest?

A

(1) Hyperthyroidism; (2) methimazole or propylthiouricil, radioactive iodine, or surgical removal.

441
Q

Why are foot ulcers such a big problem with diabetics?

A

Trauma or infection may be present, but undetected due to the loss of sensation and pain. It is worsened by vascular insufficiency and the decreased ability to heal.

442
Q

What is HHS

A

Since a type 2 diabetic is still producing some insulin, they will not present with DKA. Instead, it is called hyperosmolar hyperglycemic state (HHS) or non-ketotic hyperglycemic hyperosmolar syndrome (NKHS). It usually occurs with dehydration and/or infection and may present with confusion or disorientation. It is diagnosed by hyperglycemia (blood glucose > 500 mg/dL), hyperosmolarity, and dehydration, without ketoacidosis. Treatment of HHS involves correcting the hyperglycemia, electrolyte imbalances, and dehydration. The mortality rate of both DKA and HHS is high.

443
Q

What is DKA

A

Ketosis can occur in type 1 diabetes. Ketosis is when the body breaks down fat, as if it were starving, and forms ketones, or ketoacids. This leads to a type of metabolic acidosis, called diabetic ketoacidosis (DKA). Epinephrine (from stress) stimulates glucagon secretion, which increases lipolysis, gluconeogenesis and glycogenolysis. Increased lipolysis leads to increased free fatty acids (FFAs). The liver then converts FFAs to ketone bodies. In addition to the above triad (polydipsia, polyuria, polyphagia), a person with diabetic ketoacidosis may present with weakness, weight loss, tachycardia, dry mucous membranes, poor skin turgor, hypotension, nausea/vomiting, mental status changes, and sometimes shock. The ketoacids (due to acetone) can cause a characteristic fruity smell on the breath. DKA usually occurs after extreme stress, such as pregnancy, infection, or anxiety. It is diagnosed by hyperglycemia (blood glucose > 250 mg/dL), low serum bicarbonate (< 15 mmol/L), and low pH (< 7.3), with ketonemia and moderate ketonuria. Hyperkalemia is present, even with potassium depletion, due to the shift of K (and water) from the intracellular to the extracellular area. Treatment for DKA is with IV insulin, IV fluids, and electrolyte replacement.

444
Q

A 40-year-old woman presents with amenorrhea and weight loss despite increased appetite. The history and physical exam show exophthalmos, tachycardia, and warm, moist skin. Which lab abnormalities are expected

A

Increased T4, T3, and radioactive iodine uptake.

445
Q

List and describe the 3 most common microvascular complications of diabetes mellitus? How would you screen for these complications?

A

(1) Retinopathy, which are microvascular changes in the retina that can lead to blindness; nephropathy, which is kidney disease caused by damage to the small vessels that can lead to renal failure; and neuropathy, which is nerve damage that causes weakness, numbness, or pain, typically in the hands or feet.
(2) Retinopathy- annual dilated eye exams; Nephropathy- annual urine microalbumin test; Neuropathy- monofilament test to assess sensation, vascular status, and skin integrity, should be administered annually

446
Q

______ is defined as a blood glucose of <60 mg/dL, with associated cognitive impairment.

A

Hypoglycemia

447
Q

The risk factors for _____ _____ include hyperglycemia, hypertension, hyperlipidemia, obesity, altered platelet function, endothelial dysfunction, elevated fibrinogen levels, and systemic inflammation.

A

Macrovascular complications / disease

448
Q

____ ____ is the leading cause of blindness and vision loss in the US.

A

Diabetic retinopathy

449
Q

____ is an oral antidiabetic drug that does not cause hypoglycemia and has a side effect of weight loss.

A

Biguanides or metformin

450
Q

Describe 4 functions of the skeleton.

A

The skeleton acts as a framework for our bodies; it serves as a site for muscle attachment; it protects vital organs and soft tissue structures; it stores calcium and other minerals; it is the site where blood cells are produced. *note – only 4 are required.

451
Q

Name the 2 major skeletal divisions and the bones that comprise each.

A

The axial skeleton is made up of the bones of the skull, thorax, and the vertebral column. The appendicular skeleton is made up of the bones of the upper and lower extremities as well as the pectoral and pelvic girdles.

452
Q

Compare and contrast cortical bone and cancellous bone.

A

ortical bone is compact, it forms the outer shell of the bone, and the bone tissue is densely packed extracellular matrix making it rigid.

Cancellous bone is the interior aspect and trabeculae, or columns of connective tissue, form a grid like pattern that is lined with bone forming cells and filled with red or yellow bone marrow. The composition of cancellous bone gives it its weight bearing properties and allows it to be highly resistant to tensile or torsional stresses.

453
Q

List the 4 shapes of bone and give an example of each.

A

Long bones - appendicular skeleton; humerus or femur.

Short bones - wrist and hand

Flat bones - axial skeleton for underlying soft tissue and large surface area for muscle attachment; skull, ribcage.

Irregular bones - don’t fit into the other categories; vertebrae.

454
Q

Describe the structure and function of the extracellular matrix of osseous tissue.

A

The extracellular matrix of osseous or bone tissue is calcified or hardened. This feature gives bone its ability to support the weight of the body and provide protection to the soft tissue structures that lie beneath it. The extracellular matrix can be divided into the organic matrix and the inorganic matrix. The organic matrix (of bone and other connective tissue types) is composed of collagen fibers and ground substance. Ground substance, composed of proteins and polysaccharides, plays an important role in metabolic functions required for bone growth and repair. The inorganic matrix is composed mostly of calcium phosphate. It also contains carbonate, magnesium, sodium, and other heavy metals, such as lead, that may have been removed from circulation.

455
Q

Be able to name and describe the function of the 4 bones cells.

A

Osteoprogenitor cells - undifferentiated cells in the periosteum, endosteum, and epiphysial plates of growing bones. When they are stimulated by bone morphogenic proteins (BMPs) they will differentiate into osteoblasts which contribute to normal bone growth and replacing worn bone.

Osteoblasts - bone building cells, responsible for the formation of bone matrix.

Osteocytes - mature bone cells; maintain the integrity of the bony matrix and have channels that allow for the exchange of nutrients between osteocytes and vessels.

Osteoclasts - “bone chewing cells” that resorb bone by removing mineral content and organic matrix.

456
Q

Describe the function of PTH – include details about where it is produced and what triggers its secretion.

A

PTH is secreted in the parathyroid glands and serves to maintain blood calcium and phosphate levels. If blood calcium levels are low, the parathyroid glands secrete PTH to return levels to normal. PTH regulates calcium by 1. stimulating the release of calcium from bone; 2. increase renal absorption of calcium while increasing renal excretion of phosphate; 3. activate vitamin D increasing intestinal absorption of calcium; and 4. reduce blood phosphate levels.

457
Q

Describe the function of Calcitonin – include details about where it is produced and what triggers its secretion.

A

Calcitonin is secreted by the parafollicular cells of the thyroid gland. It acts in opposition to PTH by decreasing blood calcium levels. Calcitonin acts to inhibit the release of calcium from bone and reduces the tubular reabsorption of calcium and phosphate. The primary stimulus for the synthesis and secretion of calcitonin is an increase in blood calcium levels.

458
Q

describe the activation pathways of Vitamin D.

A

The activation process of Vitamin D begins in the liver where the inactive form of Vitamin D is hydroxylated to form 25-hydroxyvitamin D From here it is transported to the kidneys where it is converted to the active forms of either 1,25-dihydroxyvitamin D3 or 24,25-dihydroxyvitamin D3. Vitamin D can be taken into the body in one of two ways. It can be absorbed through the intestines by dietary intake (best sources include fatty fish, cheese, beef liver, and egg yolks), or it can be absorbed through the skin by way of ultraviolet radiation from the sun.

459
Q

Be able to describe Synarthroses and Synovial joints. List and describe the types of joints that would fall into each category.

A

Synarthroses do not have a joint cavity and allow little to no movement. They include:

Synostoses - joints that allow no movement between them and are jointed by dense connective tissue or bone; e.g. bones of the skull

Synchondroses - joints that allow limited movement and are connected by hyaline cartilage; e.g. ribs connected to the sternum

Syndesmoses - permit a small degree of movement and separated by a fibrous disc and joined by interosseous ligaments; e.g. pubic symphysis and the intervertebral joints.

Synovial joints are freely moving:

Pivot joints - rotational movement; between the cervical vertebrae.

Hinge joints - will flex toward or away from one another; knee or elbow.

Saddle joints - similar to hinge joints that allow flexion and extension, but also medial and lateral movement; carpometacarpal joint of the thumb.

Plane joints - glide for movement in multiple directions; intercarpal and intertarsal joints

Condyloid joints - similar to plane joints, gliding occurs between the bones of the joint but hte surfaces of the bones are irregular; radio-carpal joint

Ball and socket - will allow movement through all degrees of freedom; shoulder and hip.

460
Q

If the synovial membrane were damaged, would the healing process be quick or prolonged? Explain your answer.

A

Healing to the synovial membrane would occur quickly because the synovial membrane has a rich vascular supply. There are capillaries near the surface of the synovium which allow blood to escape into the synovial fluid with minor injuries.

461
Q

List the risk factors associated with developing OA.

A

Age (>70s), gender (males younger women older), excess weight, joint trauma, heredity.

462
Q

Describe the role of articular cartilage within a joint.

A

Articular cartilage works in conjunction with the synovial fluid to decrease friction between articulating bones within a joint and it works to spread forces evenly across the surface of the joint and down into the bone.

463
Q

What is the role of chondrocytes within the extracellular matrix or cartilage?

A

Chondrocytes contribute to matrix breakdown by producing matrix-degrading enzymes and they synthesize new matrix.

464
Q

Describe the role of Interleukin -1beta and Tumor Necrosis Factor-Alpha in the OA disease process.

A

Interleukin -1beta and Tumor Necrosis Factor-Alpha work to promote the destruction of the joint. This interruption to the integrity of the joint makes the chondrocytes more susceptible to damage impairing their ability to produce new collagen and proteoglycans.

465
Q

Describe the various structural changes that occur in OA and how they come to be.

A

Enlargement and reorganization of chondrocytes in the superficial layer of the articular cartilage. This is followed by edema within the cartilaginous matrix. At this point the cartilage is no longer a smooth surface, microfractures begin to form, synovial fluid fills the surface cracks, and they continue to widen. Eventually they will extend through the articular surface and into the subchondral bone. The exposed bone will thicken, small fragments of remaining cartilage will break off and become free floating in the joint cavity. The trabecular bone will harden decreasing its ability to absorb shock. Osteophytes will form. And the synovial membrane will be compromised resulting in inflammation.

466
Q

Be able to differentiate between OA and RA.

A

OA is a degenerative disease typically isolated to a single joint.

Rheumatoid arthritis is an autoimmune disease which is systemic and characterized by joint inflammation caused by an immune response that leads to synovial inflammation and destruction of the joint architecture. There is a genetic factor to it - IgRF. RA has symmetrical or bilateral presentations. Average onset age for RA is younger than OA. And RA is a chronic disease that is progressive and has periods of flare ups and remissions. RA patients will complain of morning stiffness in affected joints that lasts longer than 30 minutes. RA joints loosen with movement and range of motion.

467
Q

List and Describe the 3 treatment interventions for OA.

A

Physical rehab (manage progression with strengthening exercises), oral medications (NSAIDs, corticosteroids to reduce pain and inflammation), surgical intervention (arthroscopic debridement to improve joint alignment and reduce osteophytes, knee and hip replacements).

468
Q

What is Bone Mass Density and how does it relate to the risk for developing osteoporosis?

A

Bone Mass Density (BMD) is a measure of the amount of minerals (mainly calcium and phosphorus) contained within a certain volume of bone. BMD will increase steadily through childhood and adolescence. Peak BMD is reached in the young adult years. This peak value can determine one’s risk for later developing osteoporosis.

469
Q

How does estrogen contribute to BMD?

A

Estrogen increases the production of OPG and in turn lowers the rate at which osteoclasts are formed. The decline in estrogen levels that occurs with menopause leads to a higher rate of osteoclast activity as OPG levels also decline.

470
Q

What is the relationship between physical activity and senile osteoporosis?

A

Decreased physical activity leads to an increase in risk for senile osteoporosis. As we age we become less active and so there is a decrease in mechanical loading and so osteoblast replication slows down, leading to bone growth and remodeling.

471
Q

Be able to determine whether a T-score is normal, indicative of osteopenia, or indicative of osteoporosis.

A
  • 1.0 and above is normal
  • 1.0 - -2.5 is low bone density; osteopenia
  • 2.5 or lower is osteoporosis
472
Q

Be able to identify risk factors for osteoporosis and how they contribute to diagnostic testing.

A

Screenings should be done in women (65+), men (70+), those with a broken bone after 50, people of menopausal age with risk factors, postmenopausal under age 65 with risk factors, men age 50-69 with risk factors.

Also people with a history, small bone structure, sedentary lifestyle, calcium deficiency, excessive alcohol/caffeine intake and smoking, or take drugs or have disease that relate.

473
Q

List the pharmacologic treatments for osteoporosis and describe their mechanism of action.

A

(1) Estrogen and SERMs (selective estrogen receptor modulators) (2) Bisphosphonates and (3) Calcitonin. Estrogen is very effective at reducing the progression of osteoporosis; however, risk/benefit analysis is necessary due to secondary health problems that can arise with the use of hormone therapy. Raloxifene is a SERM that has shown to be effective in the prevention and treatment of osteoporosis in postmenopausal women. Bisphosphonates including alendronate, risedronate, and ibandronate, are the most commonly prescribed drugs used to treat osteoporosis. They work to inhibit bone resorption by reducing osteoclast activity. They are effective at reducing the risk of fracture. Calcitonin works to inhibit osteoclast activity. Teriparatide is a form of parathyroid hormone used to treat osteoporosis. It stimulates bone remodeling by increasing osteoblast activity.

474
Q

Differentiate between primary and secondary gout.

A

Primary gout is characterized by the overproduction or the underexcretion of uric acid. The cause is unknown but is often associated with diet and/or alcohol overuse or a metabolic syndrome. Secondary gout is typically caused by medications or certain conditions that result in hyperuricemia.

475
Q

If a patient has a uric acid serum level of 7.2 mg/dL, are they given a diagnosis of gout?

A

No because not all people with hyperuricemia have gout (20%). They would have to do an analysis of synovial fluid for urate crystals.

476
Q

Be familiar with the 4 pathways leading to elevated serum uric acid levels.

A

There can be an overproduction of purines, a decrease int he salvage of free pruine bases, an increase of cell turn over resulting in the break down of nucleic acids, and a decrease int he amount of uric acid excreted from the body.

477
Q

Be familiar with the 4 phases of gout and the progression from one phase to another.

A

Asymptomatic hyperuricemia. Acute gout arthritis which includes pain that may last days to weeks. When the attacks subside they are in the inter-critical phase of gout. They may have more attacks and with more frequency, that’s when permanent joint changes occur and they are in the chronic tophaceous gout phase.

478
Q

List 2 ways that a definitive diagnosis of gout is made.

A

A diagnosis can only be made by determining the presence of urate crystals in the synovial fluid or in tissue sections of tophi if present and through the collection of a 24-hour urine specimen to measure whether uric acid is being overproduced or under-excreted.

479
Q

What would a patient typically exhibit on an initial exam if you suspect they have OA

A

A capsular pattern and crepitus with movement

480
Q

How does cortical bone receive its blood supply

A

Arteries branching inward from the periostel arteries and arteries branching from the medullary cavity. Cacellous bone receives its blood supply via diffusion through the endosteal surface of the bone and the canliculi.

481
Q

What drug is most commonly prescribed to treat ostoeporosis

A

Alendronate

482
Q

What cells are stimulated by PTH

A

Osteoclasts

483
Q

What medication is used to maintain normal levels of uric acid

A

allopurinol

484
Q

Damage to the synovial membrane resulting in non-specific inflammation occurs in which stage of OA?

A

Late Stages

485
Q

Articular cartilage receives its blood supply and nourishment from what?

A

Constant bathing of synovial fluid

486
Q

Postmenopausla Osteoporosis is characterized by what

A

low levels of OPG and fractures are mostly seen in the vertebrae and distal radius

487
Q

What contains the nerve and blood supplies for the osteon?

A

Haversian canals

488
Q

What is a capsular pattern

A

A capsular pattern is limited range of motion in a pattern specific to joint, indicative of a degenerative process

489
Q

If someone loses their balance when walking on uneven terrain, explain how tendons and ligaments work to protect the joint and structures within it.

A

The tendons and ligaments of joints serve in proprioception (the awareness of ones’ position in space or movement of the body). When these structures undergo stretch or torsional strain, these proprioceptive nerve fibers will cause a reflexive response to adjust the tension on the muscles (to maintain balance and not fall over) that support the joint protecting the capsule and other joint structures.

490
Q

A patient presents with serum uric acid levels of 7.2 mg/dL. They have no active inflammation or complaints of pain. Which phase of gout are they in? Explain how you came to this conclusion.

A

The patient is in the asymptomatic hyperuricemia phase of gout. They have high serum uric acid levels - 6.8 mg/dL are the threshold. However, they are not yet symptomatic. If they had progressed with gout, the next phase would be the acute gout arthritis phase and they would be expereincing acute pain and swelling.

491
Q

Give 2 reasons that males are at a decreased risk for developing osteoporosis than females.

A

Males achieve 8-10% greater peak BMD than females do; they do not go through a midlife decline of sex hormone production.

492
Q

A 60-year-old, obese female works out of her home and rarely walks due to the sedentary nature of her job. Give 2 reasons why she is at increased risk for developing OA.

A

She is a female over the age of 55; she is obese, and she is relatively sedentary which contributes to the decreased nourishment of the joint that comes with weight bearing and range of motion.

493
Q

Compare and contrast the blood supply of cortical bone and cancellous bone.

A

Cortical bone has a direct blood supply. Nutrient and Perforating arteries form an anastomosis (collateral circulation) that circulates through the bone through the Haversian and Volkmann canals. Cancellous bone does not have a direct blood supply. It receives its blood supply through diffusion from the endosteal bone surface extending outward through the canaliculi.

494
Q

A 22-year-old competitive gymnast has experienced amenorrhea for the past 5 years. Her PCP is suspecting that she may have premature osteoporosis. Would you expect her RANKL levels to be high, normal, or low? Explain why her levels would be at this level.

A

Her RANKL levels would be higher than normal. Amenorrhea results in lower estrogen levels. Estrogen increases the production of OPG which inhibits RANKL. Lower estrogen levels would lead to lower OPG levels in turn increasing RANKL and osteoclast activity.

495
Q

A 68-year-old, white Caucasian female has a DEXA scan at the recommendation of her PCP. Her scan yielded a T-score of -2.6. Interpret the results of her DEXA scan. Develop a treatment plan giving a pharmacologic recommendation if applicable (be specific with name of supplement and/or class of drug), and a non-pharmacologic recommendation.

A

This T-score indicates osteoporosis. This patient should take a Calcium and Vitamin D supplement. In addition, she should be prescribed one of the following: Estrogen, SERMs, Bisphosphonates, or Calcitonin. She should also participate in regular weight bearing and resistive exercise being careful to choose activities that would not put her at increased risk for falling.

496
Q

A 72-year-old male is said to be in phase 3 of gout. He is obese and has a history of alcohol abuse. Develop a treatment plan including specific pharmacologic intervention and a non-pharmacologic recommendation to manage his disease.

A

Phase 3 of gout is called inter-critical gout. The patient is asymptomatic, and no joint abnormalities are present. The goal of treatment in this phase is to maintain normal uric acid levels and prevent progression of the disease. Allopurinol is a prescription drug that is used to reduce uric acid levels. This patient should be encouraged to lose weight and decrease his alcohol consumption. He should also avoid purine rich foods such as fish, bacon, and liver.

497
Q

A patient has a bilateral presentation of pain in the PIP joints of the hands. You believe this pain is due to arthritis but are unsure whether it is a result of RA or OA. What are 3 questions you could ask this patient to help you differentiate between RA and OA?

A
  1. Do you experience stiffness in the morning? 2. If so, does it last less than or longer than 30 minutes? 3. Does prolonged movement aggravate or alleviate your symptoms? 4. Do you ever have periods of relief, or do you experience pain daily? 5. Did your pain start in both hands or did it start with one hand and develop in the other over time? 6. What is your profession? (occupations requiring repetitive movements often lead to OA)